Этого треда уже нет.
Это копия, сохраненная 20 июня 2019 года.

Скачать тред: только с превью, с превью и прикрепленными файлами.
Второй вариант может долго скачиваться. Файлы будут только в живых или недавно утонувших тредах. Подробнее

Если вам полезен архив М.Двача, пожертвуйте на оплату сервера.
Тред тупых вопросов №101 ARISTARCHUS EDITION 456720 В конец треда | Веб
Тред вопросов о жизни, Вселенной и всём таком.

Спрашиваем то, за что в других местах выдают путёвку в биореактор. Здесь анонимные учёные мирового уровня критически рассмотрят любые гениальные идеи и нарисованные в Paint схемы.

Предыдущий тут: >>451413 (OP)
https://2ch.hk/spc/res/451413.html (М)

Q: Можно быстрее?
A: Можно упасть в пузырь альбукерке, наса уже почти надула его.

Q: Я начитался охуительных историй про уфологию, че делать, нам жопа?
A: Да, тебе жопа, можешь сгонять в зогач или куда оттуда пошлют.

Q: Что будет с человеком в вакууме без скафандра / если он упадет на черную дыру / попробует ступить на поверхность газового гиганта/солнца?
A: Он умрёт.

Q: Почему бы не привязать ракету к воздушному шару или стартовать с горы?
A: Космос - это не как высоко, а как быстро, большая часть энергии ракеты уходит на разгон вбок.
Подробнее тут https://what-if.xkcd.com/58/ (английский) https://chtoes.li/orbital-speed/ (перевод)
2 456724
>>6720 (OP)
Ебанный стыд...
Во-первых, Алькубьерре.
Во-вторых, не упасть, а создавать вокруг корабля изнутри (иначе кина не будет).
В-третьих, НАСА искривляет пространство на десятимиллионную часть, контролируя это сверхточными интерферометрами, до самого варп-привода здесь - как до Антарктиды раком.
3 456731
>>6724
Я спокоен, теперь это легитимный тред.
8768657.png29 Кб, 980x643
4 456733
Дано: есть качели-подобный механизм, у которого есть Плечо1, Плечо2 и центр, разделяющий плечи, приподнятый на Высоту. Анон стоит на П2. На конец П1 со скоростью V1 под углом У падает астероид/метеорит. Анона возьмём массой 100кг для ровно счёта и представим, что он не по-мясному упругий, а монолитно твёрдый, как кусок металла.
Вопрос: Какими должны быть параметры П1, П2, В, У и V1, чтобы такая установка V2 стала достаточно, чтобы смогла благополучно доставить анона в космос/на околземную орбиту/сделать искуственным и временно живым спутником планеты/отправить за пределы солнечной системы?
5 456734
Интересные ученые, к сожалению, начинают заканчиваться, на следующую пару тредов материал еще есть, но потом туго придется.
6 456735
>>6733
Орбита это вбок, а не вверх. Если анон полетит вертикально вверх на скорости меньше второй космической, то он один хуй потом опять на земляшку ебнется,
7 456736
>>6734
Ничего они не заканчиваются, просто посмотри на названия крупнейших кратеров на Луне и сразу же найдешь ученых-гигантов, которые просто ускользнули от твоего внимания. Я таким образом авторов-классиков находил, только ориентируясь на кратеры на Меркурии. Как базы имен ученых и авторов, лол.
8 456737
>>6733
Любые подобные качели/катапульта/пушка не годятся для запуска людей, разумных параметров для такой хуйни попросту не существует.

Во-первых, в собственно момент запуска ускорения составят тысячи g, человек, стоящий на плече, моментально всмятку. Во-вторых, даже при игнорировании перегрузок, при таком старте трение о воздух тебя сожжет и обдерет до костей. И так далее.

Для более-менее плавного запуска требуется плечо длиной в сотни километров, что очевидно невозможно.
15413255211400.jpg80 Кб, 750x1000
9 456738
>>6735
Но если он на высокой орбите хотя бы прордрищетсяпукнет, то в принципе это может придать толчка достаточного, для перехода на орбиту. Утрирую, конечно, про

> пук


но суть в том, такой толчок поможет ультрасъекономить на топливе в экстренных условиях и уже там, в космосе, потребует совсем немного энергии для выхода на стабильную орбиту или превышение второй космической
10 456739
>>6737
А если планета безатмосферная?
Сорян за пикчу >>6738, промахнулся
11 456740
Как выглядит сингулярность?
Допустим я попал в черную дыру. Забудем обо всем, что меня убьёт и т.д., так вот, перед собой я увижу сингулярность. Что она из себя представляет? Она круглой формы? Будет ли она иметь цвет?
Теоретически она же должна как-то выглядеть.
12 456746
>>6740
Ну, теоретически, ты не сможешь увидеть форму, а будешь видеть только ослепительный свет, стоя на поверхности самого ядра ЧД, ибо ВСЕ частицы "падают" с неба - в данном случае горизонта событий этой ЧД - на тебя, наблюдателя, стоящего на поверхности ЧД. И вокруг будет свет и шум всех видов - фотоны всей линии спектра, от УФ до ИК, все шумы, все флуктуации, всё сразу. Если сравнить с ПК, то твои глаза, уши и вообще восприятие испытает тотально ДДоС. И это мы не берём в расчёт, как ты указал, давление, гравитацию и вот это всё.
13 456747
>>6746

> самого ядра ЧД


Так, погоди. У дыры ещё и ядро есть? Я думал, что всё сжимается в одну точку, то есть сингулярность вместо ядра. А почему существует ядро и сингулярность не засозасывает его?
14 456748
>>6740
Никак, сингулярность же нулевого размера: у стационарной черной дыры — точка, у вращающейся — кольцо нулевой толщины.

Но и если бы это было не так, увидеть их все равно было бы нельзя, так как даже находясь под горизонтом событий, невозможно получить информацию с большей глубины.
15 456749
Способен ли российский сегмент МКС существовать самостоятельно?
А насавский?
16 456750
>>6746

> ибо ВСЕ частицы "падают" с неба - в данном случае горизонта событий этой ЧД - на тебя, наблюдателя, стоящего на поверхности ЧД. И вокруг будет свет и шум всех видов - фотоны всей линии спектра, от УФ до ИК, все шумы, все флуктуации, всё сразу. Если сравнить с ПК, то твои глаза, уши и вообще восприятие испытает тотально ДДоС. И это мы не берём в расчёт, как ты указал, давление, гравитацию и вот это всё.


Я немного перефразирую вопрос. Это и так понятно.
Если я возьму и сожму вселенную в одну точку, кругом ничто. Нет ни частиц, ни шумов ни флуктуаций.
Я допустим Бог, мне ничего не мешает смотреть на неё. Как она будет выглядеть?
Это логично, что будет шарик. Так, какого он цвета? Если весь свет он проглотил, то бесцветный? Абсолютно чёрный шарик или что?
Не верю, что никто не рисовал её как она теоретически должна примерно выглядеть.
17 456751
>>6739
Как будто это спасет тебя от безумных перегрузок. А если хочешь лететь не в виде кашки, то второй конец качели должен будет пройти нихуевое расстояние, которое полностью исключает возможность подобных катапульт в реальности.

Допустим, мы не какие-нибудь там слабаки и можем долгое время выдерживать 10g перегрузки, а вылетать из пращи мы будем со скоростью 5 км/с (остальное доберем ракетами). Тогда ускорение будет длиться 5000/100 = 50 секунд, а пройденный путь составит at²/2 = 100×50×50/2 = 125000 м = 125 км. Катапульты подобного размера это явно фантастика.
18 456752
>>6750
Это про человеческий глаз больше вопрос, а не про космос. Отсутствие фотонов = черный цвет.
19 456753
>>6750
Она будет выглядеть умозрительно, а не визуально.
Какого цвета корень из четырёх? Корени из четырёх это плюс два и минус - вот как они выглядят? А какая-нибудь математическая константа типа числа пи как выглядит, как шарик? А цвета какого, прозрачного? Чёрного?
С таким узеньким и примитивным пониманием мира, замкнутым в рамках видимой части спектра (а это ООООООООЧЕ мало в сравнении со всем спекторм) до Бога тебе ой как далеко.
20 456755
>>6753

>С таким узеньким и примитивным пониманием мира, замкнутым в рамках видимой части спектра (а это ООООООООЧЕ мало в сравнении со всем спекторм) до Бога тебе ой как далеко.


А я и не сказал, что я умный. Тупее тебя, можешь гордиться.
Но если я правильно прочитал, в ОПпосте написано "Тред тупых вопросов", вот и задал, какие претензии?
21 456757
>>6755
Я тебя не пытался именно оскорбить, просто х/з как донести, что ты слишком зажат в рамках очень-очень-очень маленькой части восприятия мира: ну не может весь ботанический сад определиться внешним видом одного одуванчика.
1548066644908.png44 Кб, 512x512
22 456758
>>6757
Ну ладно, теперь понял.
23 456764
>>456671
Я про нормальный радиус. 4 км, или сколько там должно быть по расчетам. Ну так что с силой кориолиса? Сильно она будет мешать?
24 456765
>>6724
Альбукерке смешнее звучит
25 456769
>>456766
Узнал про то, что через 2300 лет на сосаче создают треды его имени.
26 456844
>>455220
До сих пор печот и не хотят признавать первенство. Поэтому вместо "ПЕРВЫЙ ЧЕЛОВЕК" название типа "ну там типа мужик высадился куда-то. на луну вроде, хз, не слышал про такое никогда. а может не высаживался, может пиздят"
27 456848
>>456717
Просто надо летать сиськами вниз, тогда они будут не давить а свисать
28 456849
>>6749
нет
29 456850
>>6750

Чтоб что-то видеть нужны фотоны, а их у тебя нет

>Нет ни частиц, ни шумов ни флуктуаций.


Даже если бы чуть осталось на "посмотреть", то у тебя была бы абсолютно черная дыра на абсолютно черном фоне. То есть ты бы нихуя все равно не увидел, даже если бы фонариком светил
30 456869
>>6849
Чому?
31 456900
>>6869
Потому что станция пилилась совмесно. Если вы с друганами васянами и колянами идете на природу и хотите сделать бутеры вместе, то один берет хлеб, другой масло, третий колбасу. Но самостоятельно вы бутеры не сделаете, если не встретитесь.
Это не вписка автономных станций, а каждый модуль имеет свое назначение. Отстыкуешь модуль с электричеством - все сдохнут. Отстыкуешь модуль с СЖО - все сдохнут. Отстыкуешь модуль с сортиром - все сдохнут.
32 456918
>>6900

> Отстыкуешь модуль с сортиром - все сдохнут


Срут в памперсы и складывают в скафандры
33 456937
>>6765
Манькубьерре еще смешнее
34 456938
>>6844

> не хотят признавать первенство


Тут даде не первенство, а единство. Мы ведь даже не вторые, мы никакие. Так что сомневаюсь, что причина надмозгового перевода в этом
35 456943
>>6749>>6900
Сегменты технически независимы. Есть электрические связи через трансформаторы и передача данных, само собой, но и только. СЖО, СОТР, системы связи сегментов по отдельности полноценны и достаточны. В случае расстыковки российскому будет не хватать энергии, а американскому придётся полагаться только на гиродины для ориентации и транспортные корабли для коррекции орбиты.
36 456967
Черный дыра засосет солнце или солнце сожгет дыру?
37 456971
>>6967
Первое.
38 456982
>>6971
А если я туда попаду меня вероятнее всего распидарасит или я окажусь в другом мире?
39 456983
>>6971
Если стрельнуть самой маленькой черной дырой в звезду типа Солнце со скоростью половины световой что произойдет?
40 456995
>>6982
Тебя распидорасит ещё до того, как ты туда попадёшь.
41 457001
>>6995
Тоесть все что туда попадает тому пизда?
42 457005
Как думаете человечество сможет победить черную дыру? Например пустить в нее много ядерных боеголовок.
43 457019
>>7001
Да, всему чему угодно пизда, причём даже не когда это что угодно туда попадёт, а ещё на подлёте - его разорвёт приливными силами.
>>7005
Бля, нет, и уж тем более не так. Ты думаешь что можно как-нибудь навредить объекту, который может спокойно распидорасить Солнце, а потом всё так же спокойно "сожрать" остатки?
44 457034
>>6983
Купи universr sandbox, да проверь. Че ты как маленький?
45 457036
>>7019
Даже предположить немогу что эту хуйню может остановить, если оно может сожрать солнце.
46 457037
>>7036
Спойлер: другая чёрная дыра. Правда они просто сольются в одну большую.
47 457041
>>7037
Да это понятно что они станут одной. Просто я представил ситуацию что на нас надвигается черная дыра. И хочется предположить какие меры может предпринять человечество перед таким пиздецом. Максимум я думаю траекторию поменять, но наверно вряд ли есть сила которая ее может сдвинуть
48 457054
>>7041
У нас будет только два варианта - либо съебать, либо выпилиться.
49 457055
>>7054
Нахуя? Думаешь смерть будет мучительной?
50 457056
Что если все ядерное оружие Земли скинуть на Солнце?
51 457058
>>7056
на земле не останется ядерного оружия
52 457062
>>7056
Да вообще нихуя. Ты сравни размеры Солнца с размерами Земли - у него даже пердёж больше этой планеты в 10 раз. Забудь ты вообще про ядерное оружие, это на Земле оно что-то решает, космосу поебать на маленький шарик плазмы, там и так гигантских шаров плазмы дохуя, начиная с Солнца.
53 457063
>>7058
Солнце станет единственной ядерной сверхдержавой и будет держать Землю в страхе. Хотя оно и так по сути
54 457076
>>7056
Даже если все ядерное оружие Земли скинуть на Землю, ничего особенного не будет. Ну вымрут там всякие людишки (и то не факт), куча пыли в атмосфере, может быть на годы. И все. Ничего не расколется и не сойдет с орбиты.
56 457082
>>7077
И че? Ну да, при определенной мощности этого можно достичь. Но суммарная мощность всего ЯО на текущий момент даже не близка к этому.
57 457083
>>7041
А вот черная дыра ведь может распидорасить звезду в прямом смысле, вырывая из нее вещество приливными силами.
Вопрос. А большая черная дыра может распидорасить маленькую?
58 457087
>>7083
Нет. ТОлько поглотить
59 457090
>>7083
Просто дырка в дырке окажется.
60 457143
>>7090
Горизонт событий внутри горизонта событий? А будет ли мелкая черная дыра изнутри всасывать вещество большой черной дыры?
А ещё я подумал. А что, если наша вселенная - это внутренности черной дыры? Все сходится ведь. Сингулярность в самом начале, все дела.
sage 61 457144
>>7143
>>7090
Вы тралите?
Во-первых, Алькубьерре. Черные дыры сольются горизонтами и объединятся в ЧД побольше, с излучением гравитационных волн, что уже неоднократно наблюдалось.

>Сингулярность в самом начале


Там другая сингулярность.
7.2-7.3.png12 Кб, 370x330
62 457170
>>7083 >>7090 >>7143
"Если лучи света, образующие горизонт событий, т. е. границу черной дыры, никогда не смогут сблизиться, то площадь горизонта событий может либо оставаться той же самой, либо увеличиваться со временем, но никогда не будет уменьшаться, потому что ее уменьшение означало бы, что по крайней мере некоторые лучи света на границе черной дыры должны сближаться. На самом деле эта площадь будет всегда увеличиваться при падении в черную дыру вещества или излучения (рис. 7.2). Если же две черные дыры столкнутся и сольются в одну, то площадь горизонта событий либо будет больше суммы площадей горизонтов событий исходных черных дыр, либо будет равна этой сумме (рис. 7.3). То, что площадь горизонта событий не уменьшается, налагает важное ограничение на возможное поведение черных дыр."
63 457178
Чем топливо отличается от окислителя?
64 457180
>>7178
Всем. Окислитель это то, что даёт возможность топливу гореть.
65 457182
>>7180
Может это топливо дает возможность окислителю гореть.
66 457184
>>7182
Учи химию
67 457185
>>7178
Тем, что окислитель содержит кислород, чистый или в соединении/смеси с чем-то. По определению, горение - быстрый процесс окисления, потому то, откуда кислород берется, называется окислитель, а то, к чему он присоединяется - топливо. Ну или есть еще монотоплива, которые в одной бочке содержат все что надо.
68 457186
>>7185
А если вместо кислорода фтор?
69 457188
>>7186
Будет офторитель.
70 457190
>>7188
Чому ? Окислитель ведь от слова кислый
71 457213
Что будет если на орбиту вместо людей запустить карликов? Понадобятся ли вдвое меньшие орбиты?
72 457218
Если две чёрные дуры столкнутся лоб в лоб, а не крутясь друг вокруг друга, насколько слабее будут гравитационные волны и чё вообще можно будет увидеть?
73 457219
>>7213
Конечно. А если карлик едет из Москвы в Питер, то ему надо проехать не 700 км, а всего 350.
15438727077011.jpg338 Кб, 922x930
74 457220
Поглотит ли маленькая чёрная дыра самую массивную звезду, например R136a1? Читал где-то что самая маленькая чёрная дыра массивнее самого большого пульсара, так что пульсары не берём в расчёт.
75 457221
>>7220
Что ты понимаешь под словом поглотит? Черные дыры обычно звезды не поглощают, а распидорашивают. Большая часть вещества тупо улетает.
А большие звезды распидорить легче, у них плотность маленькая.
76 457241
Аноны, я возможно долбоёб, а возможно все заключается в том что я с телефона гуглю, но никак не могу найти координаты пустоты Эридана. Кроме упоминаний о, собственно, созвездии Эридана ничего не вижу. Может поможет кто?
77 457255
1. Если в какое-то небесное тело врежется какое-нибудь другое небесное тело - если я буду в корабле неподалёку, услышу ли я что-нибудь?
2. Частицы от удара смогут долететь до корабля или нет?
78 457261
>>7255
1. Если вне атмосферы, то нихуя не услышишь, если, правда, тебе не прилетит осколком.
2. Могут долететь, а могут и не долететь.
79 457264
>>7241
координат не нашел, но почему там пусто?
image.jpeg300 Кб, 800x600
80 457267
>>7264
Потому что там ничего нет
81 457272
>>7267
как же так
ХРД.jpg1,5 Мб, 1452x3084
82 457282
>>7178
Не слушай этих неграмотных >>7180>>7185
83 457307
В космосе можно выстрелить из пистолета, а потом догнать пулю на космическом корабле, облететь её с разных сторон, потрогать и как самурай ударить по ней, отбив в сторону?
84 457308
>>7307
Разрешаю.
85 457317
Возможно ли в теории связать два объекта в космосе тросом? Например Луну и Землю. Возможно ли существование материала который выдержит это?
И если да, то как изменятся их траектории в будущем?
425245.jpg38 Кб, 740x460
86 457322
У меня мой тупой вопрос. Представим гипотетическую ситуацию, что на расстоянии примерно половины пути от Земли до Луны внезапно появляется планета размером с Марс или чуть меньше. Не важно откуда, телепортировалось из соседней мультивеселенной. Примерно такой же плотности и массы как Земля/Марс. Следует такой же траектории, постоянно находясь между Землей и Луной, не сближаясь и не удаляясь, физических столкновений нет. Можете примерно почувствовать что будет дальше? Какие изменения на Земле произойдут в первые минуты/полчаса после этого события? Какие в первый день, первые дни, недели, месяцы?
87 457323
>>7322
нас с луной слегка распидорасит
88 457324
>>7323
Я и так догадываюсь об этом. А за одно пидорасить начнет всею Солнечную систему. Но мне интересно как это будет происходить конкретно в каждый момент времени и с какой скоростью, какие процессы уже с первых минут этой "телепортации", а не в целом типа станет немножко хуево.
89 457326
>>7324
Есть такая программа называется Universe sandbox 2 - купи на торрентах и можешь там экспериментировать, швыряя хоть черную дыру в солнечную систему и смотреть что будет.
90 457327
>>7324
ну помнится был симулятор простой там создаешь систему стабильную как наша, а после добавляешь объект с заданной массой.
(анон выше посоветовал сандбокс, а тот работал в браузере и был очень элементарным)
91 457329
>>7326
Спасибо, не слыхал о такой программе.
92 457331
>>7317

>Возможно ли в теории связать два объекта в космосе тросом?


Разумеется. Например, МКС и космонавта, который работает снаружи.

>Например Луну и Землю.


Нет. Начнем с того, что трос должен быть пиздец каким прочным, чтобы выдерживать свой вес. Расстояние между Землей и Луной меняется, так что трос должен еще и растягиваться процентов на 10. Но хуже всего - Земля-то вращается, так что трос просто начнет на нее наматываться и в итоге порвется.

>Возможно ли существование материала который выдержит это?


Если убрать вращение, наверное, теоретически возможно. Трос настолько прочный, чтобы остановить вращение, стопудово невозможен. Да и это вызвало падение Луны на Землю.

>И если да, то как изменятся их траектории в будущем?


Хз, но не принципиально.
93 457333
>>7331
хорошо, тогда возможно ли приблизить Луну к Земле, что бы орбита оставалась стабильной? И если да то на сколько близко?
94 457335
>>7322
Это невозможно, если "магию" ограничить одномоментной телепортацией. Тело между Луной и Землей не сможет двигаться с той же угловой скоростью, что и Луна, оно должно двигаться гораздо быстрее (ну или быть оснащено постоянно действующими двигателями).
95 457337
>>7333
Да. Собственно, Луна потихоньку удаляется от Земли, так что во времена динозавров она была ближе, чем сейчас. А миллиарды лет назад - намного ближе. Насколько можно приблизить не скажу, но подозреваю, что до предела Роша: дальше Луну просто порвет приливными силами, а до этого ничего, по идее, мешать не должно.
96 457344
>>6720 (OP)
Почему все планеты имеют имена римских богов а Уран греческого?
97 457345
>>7344
Чтобы пиндосы могли смищно шутить про твой анус.
98 457346
>>7255
Неподалеку - это насколько неподалеку? Вообще звук от взрывов в вакууме будет слышно вблизи, потому что ударная волна из продуктов взрыва все равно никуда не девается.
99 457353
анон,поясни: когда Чэлленджер влетит в туманность Аорты сможем ли мы наши праправнуки получать какие-либо данные? Сам я ни в зуб ногой в космосе, но где-то слышал об этом (о факте вхождения аппарата Чэлленджер в туманность Аорты).
100 457365
>>7353
Возможно ты имел ввиду Вояджер и Облако Оорта
101 457366
>>7365
Да, именно это. Немного перепутал названия.
Sage 102 457383
Как хорошо распространяются радиосигналы в космосе? Везде вижу противоречивые данные. С одной стороны говорят что все сигналы которые были выпущены с Земли со времен создания радио, уже за пределами солнечной системы превратились в шум, с другой же SETi к примеру,
ch3d.jpg75 Кб, 655x493
103 457435
>>7353

>когда Чэлленджер влетит в туманность Аорты


В шапку следующего треда, блеать
104 457438
>>7366
Скорее всего нет, ему осталось жить несколько лет.
105 457447
>>7383
Земные радиосигналы хорошо видны, но только на небольшом расстоянии - несколько световых лет. То есть первые радиосигналы 150 или сколько там лет назад сейчас хуй разберешь. В целом можно, но нужна антенна с огромной площадью, направленная точно на Землю.
Собственно, по этой причине СЕТИ хуета - антенны у них говно и они не слушают конкретные точки, а в пределах "хорошей слышимости" точно ничего нет.

Но очень мощные радиосигналы от квазаров и прочих хорошо слышны.
106 457459
Возможна ли коммуникация при помощи гравитационных колебаний? Как далеко распространяться волны?
DIOJOPreparingfight.png1,3 Мб, 710x654
107 457509
>>7322
Если убрать законы небесной механики, то тогда на Земле будет катаклизм на катаклизме, так как вода мирового океана и воздух в массе своей стянется бугром к тому месту, где находится этот Как-Марс. И если Как-Марс будет крутиться на орбите Земли, то и эта огрмная пупырка из воды, и воздуха буде вспучвать поверхность огого как. А ведь ещё эффект подъёма коры, ну тут уже сам угадай, что будет.
108 457519
>>7447
А радары в космосе будут работать?
109 457524
>>7519
Почему будут? Они и так уже работают.
110 457532
Почему границы для изучения вселенной так медленно расширяются?
-57904538236376750.pdf-28[1].jpg33 Кб, 720x540
111 457536
Свет в видимой части спектра от фиолетового до красного нам виден, а рентгеновский, ультрафиотетовый, инфракрасный и радиволновой - нет. Почему? Разве фотон, вылетевший из какой-нибудь рентгеновской звезды уже не фотон? Я запутался. Всё что далеко фиолетовее света - радиация, ад и погибель, там не фотоны, а всё что далеко краснее - звук и волны. А откуда тогда берётся фотоновый свет?
112 457537
>>7536
Просто наши глаза настроены под детект только определенного вида спектра, который мы называем видимым светом.
113 457538
>>7536

>Почему?


Потому что фотоны взаимодействуют с молекулами-атомами рецепторных клеток. В зависимости от частоты, взаимодействуют по-разному, вплоть до того, что не взаимодействуют никак.

>там не фотоны


Фотоны.

>всё что далеко краснее - звук


Ну это вообще пиздец.
114 457542
>>7538
проиграл с пиздеца, звук блять
115 457545
>>7538
>>7542
Так люди, не знающие разницы между упругими и электромагнитными волнами, это ж и есть целевая аудитория треда.
116 457550
>>7545
да про звук даже дети знают
117 457553
Самый фановый тред на спейсаче.
118 457556
Вопрос не имеет отношения к спейсачу, но коли здесь сидит мировая научная илита, то спрошу именно тут. Имеем 2 цилиндра с водой, высотой 1 метр на поверхности земляшки. Один диаметром 1м, другой 2м. Давление воды на дне этих цилиндров ведь будет одинаковым?
119 457559
>>7556
Рассмотри вопрос иначе - представь, что ты в бассейн устанавливаешь цилиндры без дна, с разными диаметром. Думаю ты уже понял ответ
120 457562
>>7559
Ок, а теперь ставим эти цилиндры в кастрюли (чуть больше диаметра самих цилиндров). Какой цилиндр первым давление воды выдавит из кастрюли, при условии, что высота наполнения водой этих цилиндров одинакова?
121 457564
>>7562
кастрюли разные или одна? дно цилиндра закрыто ?
Screenshot1054.jpg93 Кб, 797x677
122 457567
>>7564
Нет, цилиндр сквозной. Кастрюли для каждого цилиндра свои, чтобы они могли в них влезть. Можно даже на цилиндры уплотнения надеть, чтобы они не сразу всплыли. Примерно как на схеме.
123 457568
>>7567
если размеры кастрюль и цилиндров пропорциональны то одинаково
124 457569
Так и не понял как дыры испаряются. Там некая частица делится и выскакивает изнутри и поэтому масса уменьшается, или снаружи рядом с дырой влетает внутрь и отгрызает изнутри кусочек?
125 457571
>>7568
Да, пропорциональны. Спасибо.
126 457578
>>7569
потому что они не могут стабильно держать свою массу
127 457581
>>7569
Короче (учти, я квантмех сам толком не знаю), вакуум такая забавная штука, что в нем могут рождаться пары частица-античастица. Чисто по фану, от балды. Но для этого нужна энергия (E=mc^2, просто так частицы не получатся), которая в данном случае берется из гравитационного перепада (это когда здесь дыра тянет охуеть как сильно, а в миллиметре дальше - тоже охуеть как сильно, но все же чуток слабее). В норме, эта пара частица-античастица просто аннигилирует, и энергия "возвращается" туда, откуда взяли. Но в окрестностях черной дыры может получиться так, что одна частица родится под горизонтом событий, а вторая - нет, причем улетает. И тогда они не аннигилируют. В итоге, ЧД получит "обратно" только энергию одной из них, а энергия второй улетит в виде излучения.

Альзо, это излучение - гипотеза, хотя и более-менее признанная. Оно никогда не наблюдалось, а гипотеза находится на стыке квантмеха и СТО, которые вообще-то не дружат.
128 457602
>>7581
ну это такое - если это гипотеза, то почему ученые верят, что микро-черные дыры в большом адронном коллайдере будут испаряться, а не упадут в центр Земли нахуй, пожирая нашу планету изнутри?
129 457603
>>7602

>ученые


>верят

130 457604
>>7602
Ну наверно потому, что даже наше родное Солнце зачастую обстреливает Землю частицами с куда большей энергией, не говоря уже о галактике и остальной вселенной. Алсо, частицы сверхвысоких энергий до сих пор являются одной из неразгаданных тайн астрофизики, ни у кого нет даже гипотез нет о том, откуда они берутся, а энергии у них на несколько порядков больше чем в БАК.
131 457608
>>7602
А как там вообще образуются чёрные дыры? Можно с пояснением и формулами плиз!
maxresdefault.jpg79 Кб, 1280x720
132 457610
>>7538
Радиоволны - это тоже фотоны?
133 457611
>>7610
Да.
134 457615
>>7538
>>7542
У меня инфракрасный звук в колонках хрипит, что посоветуете?
>>7610
А как по-твоему радиолампы светят?
135 457621
>>7615
Поставить ультрафиолетовые колонки, очевидно же.
На сама деле вопрос сложный, мне и самому, гаманитарию, это приходило в голову, только спросить боялся
136 457626
Нужно придумать изобретение всем двачем и запантентовать
137 457631
Возможно ли в теории существование астероида в форме огромного члена?
image.jpeg310 Кб, 1080x642
138 457632
>>7631
Оумуамуа комета
139 457633
>>7632
за ствол сойдет, не более
Screenshot-2018-12-24-at-17.26.03-6111.png803 Кб, 1462x874
140 457636
>>7631
2003 SD220

В декабре только эта залупа пролетала и едва в Землю не впенетрилась.
141 457637
>>7636
Спасибо, теперь это мой любимый астероид.
142 457643
>>7637 Выглядит как говно.
143 457656
>>7581
Квантмех и СТО очень даже дружат, и это называется "квантовая теория поля". Плохо "дружат" именно квантовая теория поля и ОТО. Реальность хокинговского излучения черных дыр, впрочем, никак не зависит от того, как будет разрешен конфликт этих теорий. От этого, правда, зависят некоторые тонкие особенности спектра этого излучения.
И да, есть еще эффект Унру, полностью аналогичный эффекту Хокинга, согласно которому ускоряющиеся тела нагреваются с той же температурой, которое имело бы излучение Хокинга, если ускорение свободного падения на горизонте ЧД в соответствующей формуле заменить ускорением данного тела. Температура эта очень маленькая, и наблюдать этот нагрев пока никому не удалось - неизвестен экспериментальный сеттинг, в котором он был бы заметен существующей техникой.
144 457657
Так может черные дыры теряют в массе, потому что тоже своего рода звезды? Но излучают в том диапазоне, который мы еще не изучили. Так сказать ЧД переходная ступень материи в иное состояние.
145 457660
>>7643
Удваиваю. Похоже на четвертый тип по бристольской шкале, а не на няшный хуйчик.
146 457670
>>7657
При потере массы излучение происходит в ИК-диапазоне, то есть грубо говоря тепло.
147 457676
>>7581
Так разве не должна тогда чд тяжелеть? Из нихуя рождаются 2 частицы, одна из которых падает в чд.
148 457681
>>7676
это виртуальные частицы
они в тебе и вокруг тебя сейчас появляются и исчезают
149 457682
>>7681
Ну и? Одна частица-то один хуй падает в чд.
150 457684
>>7682
а вторая которая должна ее аннигилировать остается вне ЧД
151 457688
>>7682
>>7684
Объяснение с падающей и улетающей частицами - поебенька, специально придуманная для интуитивного, эвристического научжоп-понимания "физического смысла" эффекта.

На самом деле суть в том, что Хокинг использовал для расчета эффекта квантовую теорию поля. Обыватель не знает про нее и про всякие там гильбертовы пространства, функциональные интегралы и тэпэ. Но ему нужно это как-то визуализировать. Придумали такую хуйню - пары "частица-античастица" (у античастицы при этом - отрицательная энергия, чтобы не нарушался закон сохранения энергии). У черной дыры, "поглощающей" "античастицу" с отрицательной энергией, масса уменьшается.
152 457690
>>7688
как раз для ответа на АиБ
153 457701
Допустим, нам нужно вывести космический корабль размером с боинг 747 на орбиту. На борту есть источник электроэнергии в виде компактного термоядерного реактора.
Вопрос: каким образом можно использовать реактор для этой цели и возможно ли сделать это только за его счет?
Мне пока в голову ничего не приходит, кроме как запитать какие нибудь турбины от реактора, которые будут выбрасывать струю воздуха из атмосферы.
154 457702
>>7701
ничего не получится.
либо корабль сам летит на родных двигателях, либо его выносят на орбиту, а дальше он сам
155 457703
>>7702
Не понял, почему не получится? Он не сможет чтоль разогнаться?
156 457705
>>7703
Еще нету компактного термоядерного реактора, неизвестны его габариты и мощность
157 457707
>>7705
Ну я же сказал, что допустим.
Какая тебе мощность нужна?
158 457708
>>7707
если ты допустил существование компактного термоядерного реактора питающего турбины, то какая проблема в представлении таких турбин
159 457719
Сап, спейсач. Можно ли сделать съемные части анти-радиационной защиты для скафандра (допшлем, жилет, наручи+наплечники, "штаны"? И сконструировать их так, чтобы их можно было нацепить и на обычный скафандр для внекорабельной деятельности и на аварийно-спасательный скафандр?
160 457720
Ъ.
161 457724
>>7708
Ну теоретически возможно разогнать корабль до первой космической с помощью таких двигателей?
162 457725
>>7724
турбины будут работать ровно до тех пор, пока будет что выталкивать
163 457726
>>7725
Я поэтому и спрашиваю, ёпта. Слоя атмосферы хватит, чтобы разогнаться? Или вылетишь быстрее, чем орбитальную скорость наберешь?
164 457727
>>7726
А что тебя держать собственно будет в пространстве без атмосферы? Улетишь вниз, опять будет работать пока не наберешь критическую высоту.
165 457728
>>7727
Ну так наберу первую космическую или нет в итоге?
166 457729
>>7724

>теоретически


Да. Забираем забортный воздух, греем его реактором, давление растет, при выбросе появляется реактивная тяга. Теоретически, греть можно до сколь угодно большой температуры, поэтому тяга неограничена. На практике, наверняка будут проблемы с крайне разреженным воздухом на большой высоте, а также с жаропрочностью материалов двигателя. Насколько большие - я хз, но подозреваю, что вплоть до полной нереализуемости без "магии".
167 457730
>>7729
Реактор - и есть магия.
Или ты имеешь в виду доп. магию?
168 457731
>>7730
Ну да. Я понимаю вопрос так, что магический реактор + современные ну или реалистичные технологии. В конце концов, даже компактный реактор довольно реалистичен в перспективе, пусть с нынешними технологиями и невозможен.
image.png283 Кб, 663x482
169 457733
>>7730
Ну вообще не такая уж и магия
170 457734
>>7731
Хорошо, тогда я вижу два варианта:
1) набирать скорость по пологой траектории, чтобы не вылететь за пределы атмосферы раньше, чем нужно;
2) использовать доп ракетные двигатели, которые начнут работать при достижении разреженных слоев атмосферы, а потом отвалятся.
171 457736
>>7731
Опять же, "относительно компактные" ядерные (а не термоядерные) реакторы и так есть, хоть на подводных лодках. Энергии там один хуй хоть залейся будет, лишь бы конструкция не расплавилась.
172 457737
Космач помоги пожалуйста побратски я приобрелмагическое мышление и все такое а я хочу опять наукоблядью стать дай советов мудрых
173 457738
Я бы пошел в саентач но тут лучше люди извините за оффтоп
174 457740
>>7737
>>7738
Очень легко. магия не работает. Проверь все, даже самое запрещенное. Ничего не сработает. Никакой "грани" нет. И за ней ничего тоже нет. Люди это бывшие мартышки которые научились пользоваться камнями.
175 457741
>>7736
Круто, тогда можно уже лететь. Выбрать один из вариантов >>7734 и начать собирать корабль.
176 457742
>>7740
я УЖЕ ПРОВЕРИЛ КОЕ-ЧТО И ОНО СРАБОТАЛО ТАК ЧТО ХУЕВЫЙ АРГУМЕНТ НО ВСЕ РАВНО СПАСИБО ТВОИ СЛОВА МНОГО РАЗ ПОВТОРЯЮТСЯ В МОЕЙ ГОЛОВЕ И ТАК Я ПРИОБРЕТАЮ УБЕЖДЕННОСТЬ ЧТО МОЙ МИСТИЧЕСКИЙ ОПЫТЭТО ИГРА МОЗГА ПРОСТО.
177 457743
>>7742
тогда нехер флудить здесь, для вас есть отдельный /mg
178 457744
>>7740
18:00:19
>>7741
18:21:19
>>7742
18:22:19
>>7743
18:23:19
И еще говорят, что магии нет!
179 457746
>>7744
Модераторы не в счет
180 457750
>>7737
А вот нихуя.

Если у тебя магическое мышление - тебе в принципе ничего никто не докажет, потому что ты уже все знаешь абсолютно. Кто создал вселенную и человека, что происходит после смерти, какой формы Земля, зачем палочнику стучать тебе в окно ночью. Магические "теории" как и радикальный религиозный догматизм это попытка объяснить все просто и интуитивно понятно. А на любую критику всегда найдется контр-аргумент, либо можно начать на личности переходить и объявить критикана безбожником, еретиком, итд итп.

Наука не понятна интуитивна большинству людей (а вещи вроде квантовой механики, наверно, вообще почти никому), но она фальсифицируема и опровержима и этим правильна. Мы нихрена не знаем об этом мире, но пытаемся. Вроде понимношк получается. По крайней мере больше не надо трястись в страхе от молний и приносить детей в жертву ради хорошего урожая.
181 457751
>>7435
Ахаха
182 457752
>>7447

> в пределах "хорошей слышимости" точно ничего нет.


Это тоже ценный результат, между прочим.
183 457754
>>7750

> По крайней мере больше не надо трястись в страхе от молний


Разве было доказано, что молния не попадает в человека?
184 457755
>>7754
С 2008 года в большинстве стран мира молням запрещено попадать в людей на законодательном уровне. Нарушившей молнии грозит штраф и общественные работы.
185 457756
>>7754
Ну я тип про то что можно не боятся что тебя Зевс ебнет с неба за то что ты Геру мандавошкой обозвал или какую нибудь другую хуйню)))
186 457771
Были какие то попытки создания аппаратов по типу "тарелочек". подъемная сила на основе магнитного поля. Или это все бред?
187 457772
>>7771
Беспилотные блюдечки?
188 457775
>>7772
ну да, небольшой массы способные из за разницы магнитного поля подниматься в воздух и практически бесшумно парить
189 457776
>>7756
Я после просмотра удивительных странствий Геракла и Зены так не думаю
190 457777
>>7775
А разница магнитного поля откуда берется?
191 457778
>>7777
вот это я и хотел у вас узнать на сколько это реально или полный вымысел - кататься по магнитному полю Земли
192 457779
>>7771
Будущее уже наступило https://youtu.be/ZwSwZ2Y0Ops
193 457780
>>7778
Магнитное поле Земли слабое слишком.
Но есть такая штука - левитация сверхпроводящего магнита. Но для сверхпроводимости нужны очень низкие температуры. НА Земле не прокатит, но где нибудь на Европе или в других холодных ебенях думаю это можно применять.
194 457782
>>7780
На Европе и так гравитация настолько низкая, что просто на лыжах можно километры проехать один раз оттолкнувшись. Главное в какую-нибудь ебучую линию не упасть, но если встать на эту линию, то можно как по хайвею ехать. Слева стена, справа стена - на 200 километров вперед - естественная ровная дорога. Ляпота!
195 457783
>>7782
Ну на Марсе значит.
196 457784
Могут ли в теории существовать газовые планеты, где смесь газов будет преимущественно кислород и азот?

>>7783
(а Марс без магнитного поля практически)
197 457785
а что если в нашей солнечной системе такие хреновые магнитные поля у планет, потому что инопланетяне их давно износили?
198 457786
>>7784

> (а Марс без магнитного поля практически)


А оно и не нужно. Магнитное поле сам создавать будешь.
Нужен лишь холод, чтобы левитировать.
199 457788
>>7786
Пиздуй в Антарктиду тогда.
200 457789
в Антарктиде то до сих пор тема кратера не раскрыта
201 457792
>>7784

>где смесь газов будет преимущественно кислород и азот


Земля, например.
202 457793
>>7792
я имел в виду планеты где ядро и сразу газ идет
203 457795
>>7793
А, в глаза долблюсь. В теории ничто не мешает, правда не ясно, как такие планеты могли бы образоваться. В принципе, сами по себе азот и кислород - не редкие вещества, но непонятно, куда делись гораздо более распространенные водород и гелий. Теоретически, наверное солнечный ветер от звезды может их "выдуть" как более легкие, если планета достаточно близка к звезде (но и не так близка, чтоб выдуло нахуй все, как у Меркурия). Не знаю, насколько эффективно и быстро это снижает долю легких газов. В принципе, нашли дохрена экзопланет, расположенных подходяще. Остается погуглить их составы, спектрометрия наверняка их выдала.
204 457796
>>7795
как спектрометрия экзопланет в январе 2019, ты чего. Джеймс Уэбб еще не полетел, другие крупные телескопы на Земле еще строятся. Максимум там считали очень убогенький спектр планет, которые пиздец впритык к звезде, как у мега горячих юпитеров.
205 457797
Почему млечный путь отчетливо видно, но не видно горячей середины?
206 457798
>>7797
пыль, газ, это всё дерьмо непрозрачное для видимого спектра. В инфракрасном лучше видно, сам сказал горячее.
207 457801
>>7796
Не, ну понятно, что если нет прямого наблюдения, то точный спектр не посчитаешь. Но хоть что-то же есть.
208 457802
>>7798
неужели этого дерьма настолько много, что мы не можем увидеть центр галактики?
209 457804
>>7788
Тоже вариант кста.
210 457805
>>7795
Вроде бы есть теория, что газовые гиганты в прошлом были планетами типа Венеры. Потом они все наращивали и наращивали газовую оболочку и в итоге получились газовые гиганты.
211 457806
Кстати насчет Венеры. Она ближе чем Марс, там все в порядке с магнитным полем, атмосфера плотная. В плане терраформирования ,в будущем, она ведь кандидат лучше чем красная пустыня?
212 457807
>>7806
На Марс ты можешь выйти в скафандре погулять хоть сейчас, только долететь надо. А на Венере - хуй: температура, давление, кислота в атмосфере. К тому же, "ближе" в Космосе так не работает.
213 457808
>>7806
Ты шо, совсем чтоль? Почитай хотя бы статью на вики, прежде чем такие вопросы задавать.
214 457809
>>7808
прочитана. Там плотная атмосфера из углекислого газа, который в теории можно преобразовать в кислород. А на марсе нет ничего.
215 457811
а все, вопрос отпал

>Венера не имеет магнитного поля. Причина его отсутствия не ясна, но, вероятно, связана с медленным вращением планеты или отсутствием конвекции в мантии.



странно, складывалось впечатление что раньше было написано обратное
216 457814
>>7809
А адское давление куда денешь?
217 457816
>>7814
так полагаю если часть атмосферы выпадет осадками проблема с давлением будет отчасти решена?
218 457818
>>7816
Нет. кислорода столько же останется.
219 457825
>>7818
хорошо, а если это будут не жидкие осадки, а атмосфера выпадет в виде льда?
220 457827
>>7825
Кислород надо связывать с водородом, чтобы выпадала вода. Но водород из Венеры уже давно упиздовал, так что сасай.
221 457828
>>7802
Его много, да, а еще оно горячее, но не настолько горячее, чтобы светиться в видимом диапазоне.
222 457829
ясно, остаемся без Венеры
223 457830
>>7827
Водород в составе атмосферы Венеры связан с серой в виде сероводорода, который тоже пиздец токсичный. Если атмосфера венеры выпадет, то будет не ice и не айс, будет сасайс. Всякие сероводороды, сероуглероды, нитриды серы могут образовываться
224 457843
Что будет с Землей и ее обитателями, если поместить ее на стабильную орбиту вокруг значительно более яркой звезды, скажем Веги? Считаем, что орбита подобрана так, чтобы Земля получала примерно столько же энергии, как сейчас, чтобы не сгорела нахуй сразу же.
225 457847
>>7843
Будет либо как Венера, либо ка Меркурий, только побольше.
226 457866
Не стал делать отдельный тред, но вот это довольно клёво так то

То есть ПИЗДЕЦ https://twitter.com/robertoglezcano/status/1030828463891120129

НА САМОМ ТО ДЕЛЕ

А если учесть что еще солнечная система тоже как то двигается, и галактика, то это довольно прикольно

Получается что те ребята на орбите эдакие кульбиты выделывают! С какой же реальной скоростью они перемещаются в пространстве тогда? Дохуя километров в секунду?
227 457869
>>7866
Да уж, мир удивителен, особенно если ты тупой.
228 457871
>>7866
70 000 километров в секунду так то.

Ебать.
229 457872
>>7869
Все таки тупизна и не_знание это разные вещи.

Тупой это когда думать не можешь, а не знаешь это когда можешь, но просто нет фактов, как только факты появятся, вывод не не заставит себя ждать.
230 457881
>>7866
Гиф не полная, движется не только Солнце, но и сама галактика, а далее по аналогии с Солнцем.
231 457886
>>7881
Ну ето то понятно, там вообще ебическая траектория получится, такую хуй нарисуешь, мне кажется, если нет всех данных, которые наверняка хранятся в весьма специфических местах и попросту не доступны простым смертным, таким, как вот этот вот автор
NotVortex.gif5,3 Мб, 800x450
232 457887
>>7871
>>7866

Наиболее «реальной» скоростью можно считать скорость относительно реликтового микроволнового фона, древнейшей и наиболее универсальной точки отсчета во Вселенной, и относительно него Земля движется со скоростью ~370 км/с в направлении созвездия Льва.

>>7886
Это не реальные траектории, а просто схематичное изображение, к тому же еще и неправильное. Троллить тупостью нехорошо.

На самом наклон Солнечной системы к направлению ее движения вокруг галактического центра не 90°, а всего около 60°, так что орбиты не вертикальными должны быть, а наклоненными, как на прикрепленной гифке. Вот тут изображены орбиты Меркурия, Венеры, Земли и Марса в правильной пропорции и под правильными углами, только размер самого Солнца и планет увеличен.
233 457888
>>7887
Про скорость хорошее уточнение, спасибо.

Про схематичность первой гифки то очевидно, да.
234 457893
>>7866

>Тупой это когда думать не можешь


Достаточно подумоть о своей собственной траектории с учетом движения Земли вокруг Солнца. Какие кульбиты я, мать вашу, вытворяю, летая со скоростью в 30 км/c!
235 457894
>>7888
Что будет если за МКС в космос вылить ведро воды. Она разлетится из-за давления или слипнется в шарик, как внутри самой МКС?
236 457895
>>7894
выкипит
237 457897
>>7894
Как ты ее выльешь? Подумай хорошенько.
238 457898
>>7795

>Теоретически, наверное солнечный ветер от звезды может их "выдуть" как более легкие, если планета достаточно близка к звезде (но и не так близка, чтоб выдуло нахуй все, как у Меркурия).



Ну если >>7784 говорит именно о газовом гиганте - такое развитие событий крайне маловероятно. У них довольно мощные магнитные поля, которые какбе предовтращают выдувание атмосферы родительской звездой. Но это даже не важно, потому что даже если у него не будет сильного магнитного поля, водород просто не успеет выветрится. Он намного быстрее вступит в реакции с кислородом и азотом, от чего получится вода и аммиак.

Так что короче нужно очень много кислорода и азота на старте формирования планеты, иначе получим обычный ледяной гигант или планету-океан. У протопланеты должно быть больше, чем 2 молекулы кислорода на каждую молекулу водорода, ибо H2O. Теоретически это возможно, но учитывая то что мы знаем о распределении вещества во вселенной - выглядит крайне маловероятным, на уровне зарождения разумной жизни на планете.
239 457902
>>7897
Набрать ведро воды на земле, закрыть крышкой, вылезти из звездолета в космосе и вылить.
240 457907
Почему взрывы от сверхновых теряют энергию, а не распространяются по вселенной бесконечно. В космосе же нечему задерживать материал взрывной волны.
241 457908
>>7907
как это нечему
242 457909
>>7847
Схуяли, ведь по условию Земля будет получать столько же энергии от новой звезды, как сейчас от Солнца. То есть орбита будет не в 1 а. е., а в несколько раз дальше.
243 457910
>>7907
Ну так этот материал потом собирается в новые звёзды и системы
Наша система результат 2-3 циклов сверхновых например
244 457911
>>7907
Во-первых, энергия падает как квадрат расстояния от взрыва (потому что площадь сферы взрывной волны растет с той же скоростью). Во-вторых, задерживать таки есть чему, всякие межзвездные облака газа и прочая хуйня. Даже там, где их нет, пространство не абсолютно пустое, какие-то атомы есть даже в межгалактическом пространстве.
245 457913
>>7907
В смысле "взрывы"? Электро-магнитные волны и нейтрино таки распространяются, газы же тормозятся излучением и другими газами, в конечном счете остывая и сбиваясь в холодные туманности.
246 457918
Поясните про зону обитаемости звезды?. Нахуя ее вообще придумали, если не имеет никакого практического значения? Будь у Земли атмосфера Венеры то на ней было бы почти также жарко как Венере, и наоборот, если поместить Венеру на орбиту Марса или еще чуть дальше, там было бы комфортные +20. Выходит что расстояние до звезды не значит почти нихуя, куда важнее конкретные особенности каждой экзопланеты.
247 457919
>>7918
Это сейчас не имеет значения, когда выяснили кучу хуйни, а раньше использовалось как правило большого пальца, чтобы примерно выделить зону, где может быть планета с жизнью по наличию жидкой воды. Помогает откинуть в сторону совсем ебанутые конфигурации Меркуриев и всё что за снеговой линией дальше орбиты Марса.

Но поместить Венеру на орбиту Марса не самая лучшая идея, там необязательно будет круто, сразу же будем меньше солнечной энергии и парник заглохнет, начнется вечная венерианская зима.
248 457920
>>7919

>парник заглохнет


Парник будет работать всегда, весь вопрос в балансе получаемой и отражаемой энергии. Насчет Венеры надо считать. Опять таки, греть планету изнутри можно и например наличием массивного спутника.
249 457921
>>7919

>Это сейчас не имеет значения


Имеет. Людишек-то интересуют как раз землеобразные планеты без пиздецовой атмосферы в духе Венеры (трудно делать какие-то предположения насчет жизни вообще, но по крайней мере на таких планетах, как мы знаем, жизнь точно возможна). И для них как раз именно обитаемая зона звезды важна.
250 457922
>>7921

>пиздецовой атмосферы в духе Венеры


Пиздецовость там только в температуре, была бы она там земной, то все остальное уже не проблема.
251 457923
>>7922
Так или иначе, парник только поднимает температуру. Без звезды вообще от него толку не будет. Обитаемая зона остается, просто за счет парника сдвигается дальше от звезды. Не сказал бы, что "расстояние до звезды не значит почти нихуя", просто это не все.
252 457924
>>7922
блядь, куда ты лезешь, ну, там вращение планеты такое что день длиннее года и вулканическая активность пиздецовая, всё в сере, выше в треде уже писали
253 457925
>>7920

>Парник будет работать всегда


Нет, ты не устроишь парник на орбите где атмосфера, которая образует парник выпадает льдом на шапках.

> греть планету изнутри можно и например наличием массивного спутника.


Это да, тут не спорю, но это тоже по времени ограниченные хуевины. Спутник может приблизиться и ебнуться, или отойти, изнутри тепло может остыть и гораздо раньше, чем хотелось бы.
254 457926
>>7923
Я про то что эта зона очень сильно двигается в обе стороны от звезды из-за особенностей атмосферы\наличия массивных спутников\других особенностей.
>>7924
речь не про особенности конкретно Венеры, а про зону обитаемости звезд.
>>7925

>Нет, ты не устроишь парник на орбите где атмосфера, которая образует парник выпадает льдом на шапках.


>


It depends. Планета может быть со старта разогрета, могут греть всякие гравитационные эффекты, может упасть крупное тело, чтобы хватило для запуска парникового эффекта с положительной обратной связью.
255 457927
>>7926

>в обе стороны


В обе-то вряд ли. Какой механизм природного холодильника ты можешь предложить?
256 457928
>>7926

>Планета может быть со старта разогрета


И со временем это говно выпадет на поверхность всё равно. На Земле и на Луне тоже была мощная атмосфера после столкновения, и хули? Всё говно выпало, смело, упало.

Венера вообще уникальная говнина с её вращением и вулканизмом, которое может ключевым образом влиять на климат, но люди думают, что это какое-то правило, и что такое может происходить чаще. На Венере эти ебанутые вулканы щитовые может срут постоянно из фумарол, подпитывая атмосферу, а так бы унесло её в пизденя солнечным ветром давно уже.
257 457930
>>7928
Разве Венера сейсмически активна? Почему-то всегда считал что она мёртвая как луна.
>>7927
Планета с отсутствием парниковых газов и высоким альбедо, очевидно же.
258 457938
>>7930

>Разве Венера сейсмически активна?


ну одна из главных гипотез, что это происходит. Поверхность то гигантская, видели лишь радаром. Если там что-то будет живо извергаться или просто парить каждый день, мы не увидим.
259 457939
>>7928
Когда уже пошлют венероход, ездить по вулканчикам.
260 457942
>>7939
Тут есть три стула. Первый- термоизоляция. На нем сидели советские Венеры. Самый технически реализуемый, но создать аппарат который проработает дольше нескольких часов невозможно. Второй стул- активное охлаждение, какими нибудь йоба элементами Пелетье. Сложно, нужно куча энергии и хз возможно ли даже теоретически. Третий, самый сложный, но самый правильный это разработка элементной базы, способной работать при температуре в 500 градусов. Начиная от транзисторов, заканчивая фотоматрицами и батареями.
261 457945
>>7942
А что насчёт дирижабля, возможно пилотируемого, который NASA планирует когда-то ниибаца в каком будущем отправить на Венеру в верхние слои атмосферы? Говорят, что человеки смогут работать под температуру в 30-40 градусов, а дышать обработанной атмосферой. Если такой дирижабль появится, то на нём могли бы быть какие-нибудь малые аппараты, которые смогут проработать у поверхности пару десятков минут и свалить обратно
262 457947
>>7945

>Говорят, что человеки смогут работать под температуру в 30-40 градусов, а дышать обработанной атмосферой.


Ну спроси себя, ты бы согласился быть астронавтом на таком дирижабле? Если что-то пойдет не так, ветер поднимется, или поднимется слой коррозийного газа откуда-нибудь снизу, или утечка и будешь падать в пизду вниз, пока давлением тебя не расплющит и никакой помощи не придет, никто тебя не спасет. Космос это само по себе опасное место, но эта хуйня увеличивает риск в миллион раз, опаснее чем на Марсе.
Какой-нибудь беспилотный летающий аппарат по твоему не должен справляться со скоростью? Летать в киселе, где не видно Солнца для солнечных батарей, где нет кислорода для горения топлива мммм....
263 457948
>>7947

>не должен справляться с температурой?


фикс
264 457949
>>7945
Тот анон ездить хотел, а не летать. Дирижабль тоже фантастика, потому что даже в условиях земной атмосферы сделать дирижабль который будет летать месяцами (годами) сложно.
265 457950
>>7949
Без посадки, быстрофикс
266 457951
Если единственная проблема - надёжность, то я уверен, что попытаются предусмотреть максимальное кол-во опасных сценариев. Насчёт астронавтов, многие ими становятся ради острых ощущений, потому что другого профита от этой профессии нет. Зп низкая. Людей, которые не заботятся о своей жизни хватит, я думаю. С энергией вопрос можно решить ядерным реактором на борту или РИТЭГом.
267 457952
Про ездить. Прямо сейчас журнал читаю "Популярная механика", написано о венероходе ну и слово, лол AREE. Таким был бы планетоход в мире стимпанка. Минимум электроники, минимум деталей, которые могут расплавиться/разъебаться атмосферой. Ещё и двигается на гусеницах.
268 457954
>>7952

>планетоход в мире стимпанка.


неиспытанная хуйня, которая была сделана лишь в духе - откиньте стереотипы, мыслите вне коробки, попытайтесь придти к нестандартным решениям, вон в античности делали роботов на шестеренках, и ты делай марсоход на стимпанке, марковках и гусеницах.
269 457955
>>7951

>многие ими становятся ради острых ощущений, потому что другого профита от этой профессии нет


Что за хуйню прочли мои глаза.
270 457956
>>7951

> С энергией вопрос можно решить ядерным реактором на борту или РИТЭГом.


Ты бы хоть узнал как эти ритеги работают. Что значит ритэг, а? Как расшифровывается? радиоизотопный ТЕРМОэлектрический генератор. Он сам вырабатывает тепло, чтобы тебе энергию дать, а ты думаешь что вырабатывая тепло в жопе, где это тепло повсюду, можно как-то запитать всякую хуйню, которая от этого тепла дохнет.
271 457957
>>7952
Металлы в такой температуре и в такой кислотности будут иметь совсем другие свойства чем на земле. Да даже те же самые гусеницы, это просто куски железа лол. На заре танко-тракторостроения создание гусеницы с ресурсом больше сотни километров, было очень сложной задачей, которую решали не одно десятилетие. На создание гусеницы под Венерианские условия понадобится не сильно меньше времени.
272 457958
>>7955
Я видел эту манеру письма его в /zog/ это совсем отбитый школьник кажется. Представляешь - пришел на борду и сразу в зог пошел. А сейчас нас тут доебывает, но хоть тут его кто-то читает, в /zog он пытался плосокоземлян просвещать и писал им стены текста, которые никто не читал.
273 457959
>>7951

>Если единственная проблема - надёжность


В данном случае это называется не надежность, а срок службы то или иного узла.
274 457961
>>7909
У Веги спектр другой, там всяко больше ультрафиолета и другой подобной жести. То есть если поместить Землю в такое место, где она будет получать столько же ИК излучения сколько получает сейчас от Солнца, то она все равно будет поджариваться от рентгена, и атмосферу будет сдувать веганским лол ветром.
275 457974
>>7961
Чего это она будет поджариваться? Важна же суммарная энергия, а она по условию та же, греет-то не только ИК. Вот посмотрел: общая светимость Веги около 40 солнечных, значит нужно орбиту брать в 6 с чем-то а. е. от Веги.

Другое дело что из-за веганского спектра будет меньше видимого света и больше ультрафиолета. Отсюда пострадает фотосинтез, то есть меньше жратвы - общее оскудение биосферы. И плюс возросшее (наверное, в несколько раз) УФ-облучение. Хотя УФ частично блокируется кислородом и озоном атмосферы, причем собственно УФ и создает озоновый слой.

Насчет солнечного веганского ветра - не уверен, что это проблема. Он, конечно, сильнее солнечного, но и орбита гораздо дальше.
276 457985
>>7974

>меньше жратвы - общее оскудение биосферы


Так биосферу и так перекосоебит и без этого. Год то в несколько раз продолжительней будет из-за большей орбиты, как и зима с летом. По-любому наебнется что-нибудь из-за засух и холодов.
мимо
277 457986
>>7985
Ну да, куча факторов. Времена года, меньше видимого света, больше УФ. Наверняка климат поменяется, даже если общее излучение звезды то же: влияние УФ на атмосферу, немного другое альбедо, раз уж спектр другой. Но я думаю, что даже крупные таксоны не вымрут. Будет что-то вроде крупнейших вымираний, дохуя видов исчезнет, но далеко не все.
278 457988
>>7959
Ты скозал?
279 457996
>>7902
А как она выльется? С помощью какой силы?
280 457997
>>7988
ті за єто ответишь
281 458000
>>7928
На Земле скорее всего наоборот сдуло большую часть атмосферы столкновением. А то, что имеем сейчас, результат действия вулканизма.
282 458006
>>7996
С помощью силы вытряхивания ведра.
283 458017
>>8006
Ну, если у тебя герметичное ведро, с крышкой которая так же герметично закрыта. Плюс тебе надо очень быстро открыть крышку и вытряхнуть воду из ведра, т.к. от контакта с вакуумом она испарится - не мгновенно, но быстро, порядка нескольких секунд.
Тебя ещё и оттолкнёт нехило в противоположную сторону, и ещё вращательный момент сообщит, если ты только не откроешь ведро таким образом, что твой центр массы будет лежать идеально ровно на оси ведра.

Ну и может быть немного (совсем-совсем) воды останется на днище и стенках ведра в виде льда, но это не точно.
284 458021
>>7942
Муриканцы для своей недавней солнечной АМС изобрели электронику, которая работает при плюс пятьсот по Цельсию. Так что венероход вполне реален, разве что технология есть только у одной страны и ей на Венеру похуй.
285 458022
>>7957
Какой кислотности? Серная кислота есть только в облаках в самых верхних слоях атмосферы, на поверхности в основном только углекислый газ, который при отсутствии воды можно считать условно инертным газом.
287 458029
>>8022
На венере кстати водоемы из расплавленного свинца.
288 458050
>>8029
Хорошо как раз будет перед банькой, прыгать
289 458057
Где находился луномобиль в посадочной ступени аполлонов ? Везде есть фото лм с аполлона 11, но там его ещё не начали возить с собой.
3Emgfeg.mp4837 Кб, mp4,
720x404, 0:15
290 458061
>>8057
Сбоку, в сложенном виде. Астронавты его вытаскивали просто, а раскладывался он сам.

https://www.youtube.com/watch?v=NBNhUNROV5U
291 458062
>>6720 (OP)
Очень глупый вопрос, но почему не додумались пиздит ресурсы с орбиты/близких комет/метеоров а так же луны?
292 458066
>>8062
не развита технология, добывать можно,Но пока затраты не оправданы, осваивают землю до конца
293 458067
>>8062

>не додумались


Кек, что бы без тебя делали. Потому что это настолько дорого, что ресурсов, которые бы окупали расходы, нет. А на орбите (самый дешевый наименее дорогущий вариант) нет вообще нихуя, кроме того, что мы туда вывели.
295 458070
>>8061
Удобно, спасибо.
296 458074
На какой скорости отдельные атомы межзвездного вещества будут пидорасить примерно как кувалдой космический корабль при столкновении с ними в полете? Что по этому поводу фантасты придумали, чтобы дырки не наделать?
297 458075
>>8074
фантасты по этому поводу придумали энергощиты и магнитные поля
изображение.png563 Кб, 917x466
298 458079
>>8074

>Что по этому поводу фантасты придумали, чтобы дырки не наделать?


ДИФЛЕКТОР ДИШ
299 458083
>>8062
Люксембург додумался.
300 458090
>>8083
Много уже астероидного железа добыли?
Я так и думал
301 458091
>>8090
Ты просто медленно мыслишь, чувак. В своё время нефть тоже была таким странным бизнесом. Чуваки могли купить право на разведку месторождений в какой-нибудь перди типа Сахалина или Ближнего Востока и люди им говорил "нахуя ты всрал деньги в какую-то хуйню?". Зато потом нефть находили и они становились миллионерами и их семью до сих пор живут припеваючи на рисковом решении, которое их предки сделали 150 лет назад. А что делали твои предки 150 лет назад? Были рабами крепостными и ты даже не знаешь их имен, лол, сгинули, их потомки наплевали на них и забыли, сейчас каждое следующее поколение вашей семьи небось начинает с нуля, потому что ничего не нажили.

Вот и с добычней на астероидах, кто-то первый ухватится за очень прибыльный бизнес и будет потом первым триллионером планеты из-за того что начали рано, а кто-то будет в помойке мусор перерабатывать и дышать испарениями, этими кем-то могут быть твои потомки, которые не будут знать твоего имени, потому что ты в своей жизни ничего не добьешься.
302 458092
>>8067
Что мешает присрать на какой-нибудь астероид бур, который будет медленно грызть его поверхность, перерерабатывая в порошок, который потом будет ускорять в плазменном двигателе питаемом энергией солнца, чтобы потом этот астероид вышел на орбиту земли, скруглил её и был доступен?
303 458094
>>8091
Технологий нет вообще. Бурение это еще самое простое. Как ты собираешься свои миллиарды тонн железа спускать на Землю? Единственные отработанные способы посадки - шаттл (дай бог 20 тонн, и то сомневаюсь) и одноразовые капсулы-спускалки для космонавтов, где полезная нагрузка еще меньше. Или ты собираешься металлургический завод в космосе построить? Ну так спуск его продукции будет сопоставим по сложности. Или надо эту продукцию прямо там и использовать, но блядь для чего? Не говоря уже о том, что ему нужна не только руда, а добывать прям все нужное на астероидах - сомневаюсь, что получится.

Единственное, что в ближайшем будущем может иметь какой-то смысл, и для чего просматриваются реалистичные технологии, - это добыча каких-то охуенно редких металлов (причем в самородном виде или в виде охуенно богатых руд), чтобы цена за тонну уже была неебической. Но для этого надо как-то прокачать геоастероидологическую разведку полезных ископаемых.
304 458096
>>8092
У астероида просто неебическая инерция. Если твой бур будет "медленно грызть и перерабатывать", а энергию получать от Солнца (чего милипиздрически мало), то может через тыщу лет он и справится. Это при том, что технологий двигателей, работающих на подножном говне, пока нет.
305 458097
>>8092
Например то, что нет ни двигателей, ни источников энергии такой мощности, чтобы двигать даже небольшие камешки в десяток метров диаметром? Ты, похоже, не представляешь себе необходимые энергетические затраты.
306 458098
>>8094
Ну железо это как бы один из продуктов, которые можно добывать на астероидах. можно и в космосе производство освоить. Я думаю можно освоить производство солнечных батарей. Потом можно создать на орбите что-то вроде станцию Склад-монтаж солнечных батарей. Типа прилетает поднимают корабль на орбиту и у него ограниченные маленькие батареи, которые нужны лишь для достижения станции Склад-монтаж, по достижении станции там уже монтируют солнечные батареи большего размера, которые произведены полностью в космосе. В космосе можно плавить железо за счет солнечной энергии, просто нужно что-то вроде доменных печей освоить. За счет центрифуги производим отделение металла от шлака.

Вон японцы там искусственный метеорный дождь хотят создавать, поднимая говно с Земли. лол, дебилизм же. Будь у меня хоть один астероид доступен я бы такой метеорный дождь над всей землей под новый год устроил, охуели бы. Все камни были бы правильного размера, чтобы сгорел, но результат был бы просто загляденье.
307 458100
>>8097
Энергии в космосе дохуя, дело лишь в её фокусировке. например можно использовать солнечные паруса, краску на астероиде, чтобы снизить его отражение и увеличить эффект Ярковского, а потом за счет гравитационных маневров два астероида можно вывести на курс столкновения на Земной орбите так что их осколки потеряют свои скорость ровно настолько, что образуют легко доступное и обрабатываемое кольцо вокруг земли с небольшим наклоном.
308 458102
>>8100

>эффект Ярковского


>Например, для астероида Голевка массой 210 млн тонн она составляет примерно 0,3 Н — в результате с 1991 по 2003 годы траектория астероида отклонилась от рассчитанной на 15 км.


Ну охуеть теперь, еще чуть-чуть постараться, и дело в шляпе.
309 458103
>>8100
Это все микроскопические эффекты дает, считанные метры в секунду в год, достаточные только для отклонения астероидов от траектории столкновения с Землей. Для вывода астероида на орбиту Земли нужно в сотни и тысячи раз больше энергии.
310 458105
>>8091
два ведра льда-9гелия-3 этому прогрессивному
311 458106
>>8103

>нужно в сотни и тысячи раз больше энергии.


в миллионы и миллиарды
312 458107
>>8103

> считанные метры в секунду в год


Ну вообще-то естественный эффект может быть таким, да, но его можно ускорить до нескольких километров в год, если постараться, а дальше зависит от массы астероида. Ты кажется к тому же имеешь в голове какие-то эксцентрические астероиды, которые пересекают орбиту Земли на огромной скорости, но у нас есть ко-орбитальные соседи, которых просто нужно найти. Они по сути земные трояны и им для выхода на орбиту Земли не хватает лишь толчка.

Это не проект на пару лет, но ты сам понимаешь о каких массах мы говорим и если какая-то корпорация получит полные права на добычу с астероида на такой орбите, то это будет реально триллион на орбите.
313 458108
>>8091
Зачем ты так много писал если мог просто скинуть научпоп на который ты повелся?
Кстати Рокфеллер в свое время был триллионером в пересчете на современные деньги
314 458109
>>8108
Ох лол, вам реально нужно какое-то одобрение или оформление слов анона? Если это было в научпопе, то это типа плохо было бы? А если бы это сказал маск, то для его сектантов это было бы евангелие. Смысл в том, что это просто реальность, с которой можно работать. Даже если очень медленно, по километру в год менять скорость скалы, то однажды её можно вывести на орбиту Земли и кто-то будет иметь на неё право(какая-нибудь Китайско-Люксембургская компания Рокфелера), а кто-то будет пыхтеть от зависти, визжать от страха и писать статьи, что это не честно, что этот астероид является достоянием всех, что нужно отобрать и поделить, что он представляет угрозу естественному порядку вещей, что нельзя двигать объекты небесных сфер чтобы не нарушить баланс природы созданный богом, , что нет там никакого астероида на орбите и это фейк, а железо добывают в Сибири, что астероид на орбите может быть использован как оружие и прочее прочее прочее...Но смысл будет в том, что у кого-то будут бабки, технологии и астероид, а кто-то будет лишь визжать и дрожать.
315 458110
1. Дробим астероид на куски
2. Захватываем устройством по типу петли
3. Доставляет на орбиту Земли
4. После необходимых расчетов (сколько по массе сгорит в атмосфере) производится спуск в прибрежное место
316 458112
>>8107

>но его можно ускорить до нескольких километров в год, если постараться



Каким образом ты достанешь дополнительную энергию? Ради интереса можешь посчитать кинетическую энергию астероида массой около тысячи тонн (это камешек менее 10 м в диаметре) и с требуемым изменением скорости в пару км/с, и сколько для этого потребуется кв.м.×лет солнечных панелей.

>но у нас есть ко-орбитальные соседи, которых просто нужно найти



В 1:1 резонансе есть, на той же самой орбите стабильные траектории только вокруг лагранжевых точек могут быть, а там искали уже, но нихуя не нашли.

Небольшого толчка тоже не выйдет, при приближении к гравитационному колодцу Земли астероид получит дополнительную энергию и улетит обратно в ебеня, если ты не погасишь его скорость. Несколько километров в секунду где-то найти все равно придется.
317 458113
>>8110
Угу. И цунами смывает нахуй все прибрежные города в радиусе пары тысяч километров.
318 458114
>>8113
Для этого нужен астероид массой в миллиарды тонн, а такие каменюки двигать уж точно нечем.
319 458115
>>8113
>>8114
Мы колем астероид, по анализу находим куски с наибольшим содержанием полезных ресурсов и их уже спускаем.
320 458116
>>8112

>нихуя не нашли.


Плохо искали, потому что технологии обнаружения дерьмо. Когда в строй LSST войдет обещают как минимум с десяток тел найти уже в первые годы работы.

>Несколько километров в секунду где-то найти все равно придется.


Ну ставишь двигатель и подвозишь топливо в пару пусков. Если на астероиде есть вода в глубине, то эту воду можно перерабатывать в топливо тоже.
Если бы у меня были технологии, я бы вообще изменил траекторию Апофиса так чтобы он проходил сквозь атмосферу Земли и совершил aerocapture.
321 458117
>>8115
На парашюте? А то половину разметает нахуй при ударе. Малейшая ошибка при расчетах - и ты уронишь свой "кусок" на мимоворонеж. Причем ошибки практически неизбежны. Либо ты спускаешь "почти вертикально" и относительно, но для этого надо твою байду на орбите очень сильно затормозить (чем?). Либо ты используешь атмосферное торможение, которое малопредсказуемо. И получаешь разброс от ожидаемой точки падения в тысячи километров.
322 458118
>>8091
Не надо тёплое с мягким путать. Нефть появилась и стала активно пользоваться именно тогда, когда стала нужна. А сегодня и в обозримом (для бизнеса) будущем потребности в разработке астероидов нет. Бизнес не думает десятилетиями или столетиями в вопросах профитов, прибыль нужно получать как можно больше и как можно скорее. Поэтому сегодня разработка астероидов тем и живёт - т.е. на деньги от энтузиастов, которые могут позволить себе пару сотен миллионов выкинуть на благое дело, да на ассигнования от правительств заинтересованных в стимулировании инженерных исследований (это вообще полезно - и цель сама по себе благая, и в процессе работы над каким-нибудь космическим буром запросто могут изобрести какой-нибудь метод бурения нефтяных скважин, который повысит эффективность работы на земле, научные открытия вообще дело непредсказуемое)

Что бы ковыряние астероидов было выгодно, нужно что бы прибыль от продажи добытого там сырья намного превышала расходы на обслуживание системы которая его добывает. А расходы эти огромные, если брать технологии сегодняшние или ближайшего будущего. То есть если на земле какого-то металла почти не останется, но будет на него спрос - да, будет выгодно. Или если у нас будут сверх-дешёвые SSTO которые на пердячьем пару будут летать в астероидный пояс и везти обратно пару-тройку сотен тонн минералов - тоже будет выгодно. Либо, если у нас будет рынок сбыта кроме от Земли - например какая-нибудь колония в астероидном поясе, для которой металлы/готовые изделия с Земли будут дороже чем покупка минералов у астероидокопателя + производство нужной продукции на месте. Ни того, ни другого, ни третьего нет и в ближайшие пару десятков лет не будет. Сейчас только-только обдумывают нужные технологии и понемногу готовят правовую базу для всего этого.
323 458119
>>8117

>и относительно


с небольшой скоростью
324 458120
>>8116

>Ну ставишь двигатель и подвозишь топливо в пару пусков



Для ЭРД проблема источника энергии все равно остается, для химических двигателей придется подвезти количество топлива, сравнимое с массой самого астероида.
325 458122
>>8117
мы их тормозим при помощи съемных двигателей от Илона
326 458123
>>8112
Энергия Солнца постоянно присутствует кругом нас, мы можем читерить с энергией Солнца чувак. Вот если бы мы были в глубоком космосе между звездами, то твой вопрос "откуда ты достанешь дополнительную энергию" имел бы смысл, а мы блядь вокруг этой ебаной термоядерной топки вращаемся и большая часть её энергии улетает вникуда. Просто нужно эту энергию собирать и фокусировать. Плазменные двигатели это первый такой шажок как это можно использовать. Но можно еще и лазерами баловаться, парусами. Вот берешь кароче гаусс пушку, которая выстреливает камни с астероида. Камень улетает в одну сторону, а отдача от гаусс пушки будет тормозить астероид.
327 458124
>>8120
нет никакой проблемы источника энергии, мы не обсуждаем работу на орбите Нептуна блядь, а в окрестностях внутренних планет.
328 458126
>>8124
>>8123
Говорю еще раз, сначала посчитайте необходимую площадь солнечных панелей, достаточную для придания 1000 тоннам астероида ускорения в 1 км/с в год, а не разводите свои маняфантазии про бесплатную энергию. Бесплатной энергии в космосе нет.
329 458128
>>8124

>нет никакой проблемы


Мне это напоминает обсуждения термоядерной бомбы, для которой нет теоретического предела мощности. Типа ну хули, можно значит взорвать Землю, ведь всего лишь надо построить бомбу чуть побольше.
330 458129
>>8128
кстати,а в самом деле можно? если опустить ближе к ядру
331 458130
>>8118
Вооот, готовят правовую базу ушлые люди, уже всё будут знать, смогут предоставлять свои услуги посредников и советников, базы для разработки необходимых технологий будут в Китае, а правовая поддержка в Люксембурге, деньги будут идти через два эти центра. А страны Х и Y будет сидеть у корыта и сосать БИБУ.

Добыча и производство в космосе это как раз то, что может в перспективе снизить эту конскую хуйню с ценами на любые космические проекты, так как даже на МКС полным полно тупого железа, которое можно было бы не поднимать с Земли, а производить в космосе, но мы были вынуждены платить за подъем. Как только кто-то освоит производство в космосе, то сразу же снизится цена на многие другие проекты, которые сейчас не взлетают, потому что слишком дорого. Но можно сделать 2 дорогих проекта - добыча и производство в космосе, чтобы сразу же снизилась стоимость на реализацию 100 проектов, которые без добычи и производства НИКОГДА не взлетят.
332 458131
С космоса даже чистое кусковое золото 999 пробы в виде самородков возить нерентабельно. В земле еще ресурсов на несколько тысячелетий минимум, копай - не хочу. Добыча и использование ресурсов может быть осмысленная только в том случае, если сразу на месте собирать что-то, чтобы не возить это с Земли.
333 458132
>>8129
Лови пасту.

> Ну давай прикинем порядок. Энергия гравитационной связи Земли (https://en.wikipedia.org/wiki/Gravitational_binding_energy) около 2 10^32 Дж. Это та энергия, которая нужна чтобы разорвать планету на части. Будем считать, что чтобы она треснула, достаточно 1/50 части этой энергии (хотя этого наверняка слишком мало), то есть 4 10^30 Дж.


>


> Тонна в тротиловом эквиваленте примерно равна 4 10^9 Дж. То есть, нам понадобится около 10^21 тонн в тротиловом эквиваленте.


>


> Царь-бомба имела массу порядка 25 тонн и выдала мощность порядка 50 Мт. То есть, для термоядерного заряда примерная удельная мощность - 2 10^6 тонн в эквиваленте на тонну заряда. Таким образом, гипотетическая термоядерная бомба, способная расколоть Землю, будет весить примерно 10^21 / 2 10^6 = 5 10^14 тонн.


>


> Объем Черного моря превышает 5 10^5 кубических километра. В кубическом километре миллиард (10^9) кубометров. Один кубометр воды весит около тонны. То есть, вода Черного моря весит примерно 5 10^14 тонн.


>


> Чтобы расколоть Землю понадобится термоядерная бомба, которая весит примерно как Черное море.

334 458133
>>8126

>Бесплатной энергии в космосе нет.


А что же питает МКС уже 15 лет? Какой там самый старый спутник?

Вот когда у нас будет астероид на орбите и астероидный завод по производству солнечных батарей, или лунный завод по производству батарей, то тогда будет бесплатная энергия?
335 458134
>>8133
Ты значение слово бесплатный знаешь? МКС самое дорогое сооружение человека за всю историю, каждый добытый киловатт стоит тысячи тонн долларов
336 458137
>>8134
Значит МКС может окупиться, лол. Хотя думаю она уже окупилась давным давно и уже банально приносит прибыль. Так что это всё еще бесплатная энергия Солнца, ничто на корабле на распадается, кроме конфеток и колбасы в желудке очередного космонавта.
337 458138
>>8132
Если расположить заряды в 6 разных точках, на глубине максимально приближенной к ядру, они при том будут такие же как царь бомба.
Речь идет не о расколе. неужели вообще ничего никто не почувствует?
338 458139
>>8137
>>8134
Ребят, их гринскрин и космонавты на тросах уже давно окупились)
ну и как же, забыл про рисованный черный фон со звездочками
339 458142
>>8138
Сейсмографы только, наверное.
340 458143
>>8142
чего то как то не очень, ожидал больше эффектов от этой планеты
ставлю 3/10
341 458144
>>8133

>А что же питает МКС уже 15 лет? Какой там самый старый спутник?



Гигантские панели общей площадью 2500 кв.м. и ценой в сотни миллионов долларов, которые дают достаточно энергии, чтобы в комбинации с хорошим ЭРД придать станции ускорение примерно 100-150 м/с в год.

Так что по этим данным можно прикинуть уже примерный порядок величин, на камешек 10 метров в диаметре и массой 1000 тонн для придания ускорения 1 км/с/год потребуется несколько десятков тысяч квадратных метров солнечных панелей. Панели, кстати, еще и со временем деградируют (около 1% в год), вечно они не будут работать. Окупят ли они хотя бы свое существование за это время?

>Вот когда у нас будет астероид на орбите и астероидный завод по производству солнечных батарей, или лунный завод по производству батарей, то тогда будет бесплатная энергия?



Конечно, нет. Бесплатного производства не бывает, всегда будут затраты на каждый произведенный продукт, плюс начальные вложения потребуются тоже колоссальные.
342 458145
>>8144
да я уж понял, что ты из пальца высасываешь цифры и произвольно добавляешь нули для приданий убедительности своим словам. Потому что ты не знаешь какую энергию с 1кв метра солнечной панели мы получаем, у тебя нет никаких мыслей о времени работы, лишь посасывание пальца и последующие выплевывание случайно больших чисел без вычислений.
343 458146
>>8145
а у тебя как будто точные цифры есть
344 458147
>>8146
смешно, что ты даже не попытался отрицать это, лол.
345 458148
>>8147
так я другой анон, просто смотрю ты набрасываешь говна на вентилятор
346 458149
>>8148
я тоже другой анон и просто смотрю за спором мелкобукв)) ой)))
347 458151
>>8149
просто стало обидно за человека , что он хотя бы что то приводит, немного жопу рвет ,а у тебя рассуждения уровня школьника
348 458155
>>8151
да нихуя он не приводит, случайные цифры приводит, называя орбитальную скорость инерцией и думая, что так как масса астероида имеет парочку лишних нулей в одной системе исчисления, то от них никак не избавиться, хотя плазменные двигатели, которые работают практически без выключений на протяжении долгого времени позволяют эти нули срезать.
Солнечная энергия это реально чит, как бы это обидно не звучало, может ему хочется чтобы мы рвали жопу за каждый килобайте ради какой-то пролетарской честности, но в космосе на это поебать. В космосе можно читерить - вон евреи будут скоро читерить сажая свой ландер на Луну, за счет периодических маневров в перицентре орбиты Земли. Читерство? Читерство. Там где кому-то нужна средняя или тяжелая ракета, кто-то может читануть физическими законами и сделать что-то дешевле. Также и с солнечной энергией, она всюду, её нужно лишь запрячь и сфокусировать.
349 458156
>>8155

>за каждый киловатт


фикс
350 458157
>>8145
Если не знаешь, то не пизди, цифры вовсе не из пальца. Панели МКС дают около 100 кВт электричества, на такую мощность ионники типа https://en.wikipedia.org/wiki/Dual-Stage_4-Grid дают около 1 ньютона, VASIMR около 2 (но больше топлива ест).

1 Ньютон / 400 тонн * 31557600 секунд в году = 87 м/с/год.

Теперь ты давай свои цифры доставай.
351 458158
>>8157
Гуглокалькулятор напиздел (считая в каких-то ебанутых недотоннах вместо метрических 1000 кг), на самом деле даже не 87 м/с в год, а 79, но сути дела не меняет.

У меня цифры, у тебя нихуя, кроме маняфантазий про бесплатную энергию, при этом никаких реальных способов эту энергию собрать и использовать ты не привел.
352 458160
>>8157
Даже с твоими цифрами изменение скорости астероида в окрестностях Земли за 15 лет выглядит как что-то реалистичное. С учетом того, что батареи на МКС довольно старые, то можно в будущем иметь такие же батареи складываемыми, доставить к астероиду и менять орбиту.
353 458163
В каком месте и при каких условиях на Земле будет минимальное значение гравитации?
354 458165
>>8158

>этом никаких реальных способов эту энергию собрать и использовать ты не привел.


Но то что эта энергия там разлита ты не отрицаешь, а мне выдумывать более тонкие методы добычи здесь не нужно, это то, что будут делать компании по добыче астероидов, лол. Будут это лазеры, или какие-нибудь электромагнитные катапульты, или же они вообще будут покрывать астероид пленкой, чтобы он нагревался изнутри и превращался в мини-комету с ретрогрейдным выхлопом с одной стороны. Скорость как-нибудь найдут как получить, главное что Солнце пичот.
355 458166
>>8163
Смею предположить, что в центре. Какие там условия, думаю, объяснять не надо.
356 458167
>>8165
Ты бля просто даун который не обладает пониманием квадратов и как следствие представить не может что вся эта халявная энергия размазана таким тонким слоем, что для ее сбора нужны километры площадей и даже больше
357 458168
>>8166
Офк имел в виду на поверхности.
358 458169
>>8168
Скорее всего или на Эвересте (высота - 8850 м, расположен ок. 27 градусов с. ш.), либо на Чимборасо. Последний имеет высоту 6267 м, зато почти на экваторе. Экватор дальше от центра Земли, поэтому, возможно, меньшей высоты горы хватит.
359 458170
>>8169
будет влиять тот факт, когда луна будет ближе всего к эвересту? или изменения будут незначительны?
360 458171
>>8170
Нет. Эверест по сравнению с поверхностью Земли - мелкий незаметный пупырышек
361 458172
>>8170
А, про это не подумал. Чуток будут, приливы ж оттуда берутся. Круче всего - во время солнечного затмения в зените.
362 458173
>>8169
Из Википедии:

> Сила тяжести, действующая вблизи поверхности Земли, зависит от широты места phi и высоты его H над уровнем моря. Приблизительное выражение для абсолютной величины силы тяжести в системе СИ имеет вид[8]:


>


> P = 9,780318 (1 + 0,005302 sin phi - 0,000006 sin^2 2phi) m - 0,000003086 H m



m выкидываем, нас интересует ускорение. Получим примерно так:
Эверест - 9,776 м/с^2
Чимборасо - 9,762 м/с^2

Для сравнения, на уровне моря на широте 55 - 9,823 м/с^2.
363 458174
>>8168
Гора Уаскаран.
364 458175
>>8172
Хотя сейчас прикинул, вкладом Луны и Солнца смело можно пренебречь, он меньше нанометра/с^2.
365 458178
Что мы имеем:
1.Нужно посрать
2.Взобраться на Эверест или Уаскаран
3. Луной можно принебречь.
Тогда я стану легче?
366 458180
>>8178
Для Уаскарана вот эта формула >>8173 дает чуть больший результат, чем для Чимборасо. Твой выбор - Чимборасо. Тем более, что он чуть ниже, скорее всего на него проще забраться.
367 458181
Известно, что Луна потихоньку отдаляется от Земли. Успеет ли она отдалиться настолько, что сойдет с земной орбиты, до того, как Солнце превратится в красного гиганта?
368 458182
>>8181
Во-первых, не успеет, во-вторых, как только Земля замедлится до периода обращения Луны, та перестанет удаляться.
369 458199
Появляются ли сейчас новые галактики?
Я вот, что имею в виду. Вселенная расширяется, становлясь всё больше, пополняется ли это всё новыми и новыми галактиками или же вселенная уже в принципе готова и расширяется просто так уже?
370 458200
Почему гелиевая вспышка не приводит к взрыву, ведь она чрезвычайно чувствительна к температуре, как тепло успевает так быстро отводиться? Ну точнее, ядро типа взрывается, но как бы понарошку, не разрушаясь, как так-то?
371 458211
https://youtu.be/Uwzg7SYZKF0

>когда пытаешься понять что спрашивают на дваче

372 458213
>>8200

>как тепло успевает так быстро отводиться?


Так нейтринами же.
373 458214
>>8182

>как только Земля замедлится до периода обращения Луны


А это когда будет?
374 458215
>>8214
через триллион лет примерно
375 458218
>>8199

>Появляются ли сейчас новые галактики?


Вряд ли, может если в единичных случаях: если галактики сталкиваются, какая-то часть звезд и прочей хуйни разлетается в стороны. Чисто теоретически, это может потом собраться в новую галактику (но звучит сомнительно).

>Я вот, что имею в виду. Вселенная расширяется, становлясь всё больше, пополняется ли это всё новыми и новыми галактиками


На какие шиши? Вещества-то больше не становится.
376 458221
>>8218
Спасибо.

> На какие шиши? Вещества-то больше не становится.


Я так и подумал, но не был уверен.
377 458229
>>8218
Магеллановы облака - это как раз следы прошлого столкновения нашей галактики с какой то другой.
378 458261
>>8213
Хуй знает, для нейтрин вроде теплее должно быть, когда уже железо майниться начинает
379 458295
>>8199
А сейчас - это когда? Или где?
На краю наблюдаемой вселенной это происходит и это в принципе наблюдаемо.
380 458302
>>8295
Думаю, оп имел в виду "по прошествии времени, приближающегося к возрасту Вселенной", скажем, через 12 и больше млрд лет после начала.
381 458303
Можно ли на Титане, спутнике Сатурна построить ракету из льда и использовать жидкий метан как топливо для взлета?
382 458304
>>8303
да можно, садись в ракету и улетай
383 458305
>>8304
Почему тогда титаняне не летают к нам?
384 458306
>>8305
говорят не долетели, солнце припекло, лед растаял
385 458307
Хули гравитационную линзу не построят? Там делов-то.
386 458308
>>8307
нук, расскажи нам, как ее строить?
387 458309
>>8308
Ну делаешь короче аппарат такой там чтоб тени были хуе-мое и в точку лагранжа между землей и солнцем отправляешь или типа того. Че сложно что ли?
388 458310
>>8303
Почему из льда? Это то же самое, что нам строить ракеты из камня.
389 458311
>>8309
Вам нужна коллаборация с тем, кто собрался ракеты из льда строить, отличная пара выйдет
390 458312
>>8305
Им Аннунахи сказали "не лезь бледь дебил сука ебаный", вот и не летят.
Только изредка на двачик пописывают.
391 458313
Что если скинуть термоядерную бомбу 100+ Мт на Венеру Допустим, в рамках президентской кампании какого-нить Трампа на 2й срок?
Почему китайцы купили документацию на 225 "Мрию"? Им не продали "Руслан", или хотят дешево в космос летать?
392 458314
>>8313

>Что если скинуть термоядерную бомбу 100+ Мт на Венеру


Ничего особенного, см. >>8132
393 458315
>>8313

>Что если скинуть термоядерную бомбу 100+ Мт на Венеру



Она (бомба) взорвется.
394 458316
>>8314
Колоть Венеру не надо, достаточно чтобы электорат смог с Земли невооружённым глазом ПОБЕДУ усмотрел, научные цели тоже найдутся.
395 458318
>>8316

>невооружённым глазом


Невооруженным глазом вся Венера видна как очень яркая звезда, и только. Уж если нужна зримая победа, то лучше бомбить Луну или Воронеж.
396 458320
>>8316
Ну так для этой цели и собирались в 50ых-60ых американцы ядрен батон взорвать на луне, что бы было заметно с Земли (особенно в Совке) и все охуели как они могут. Правда потом ещё немного подумали и решили что, мол, так нас навсегда запомнят как вармонгеров и империалистов, лучше там человеков отправить.

Сейчас я вот кстати думаю: если бы они все таки это сделали тогда, то как сегодня выглядела бы теория лунного заговора? Ничего не было, ядерный взрыв на луне снят в павильоне?
397 458322
>>8318
Луну сцуко опасно, а вот Венеру, заебок, её хорошо видно, а термоядерный взрыв с другой планеты ещё никто не наблюдал. Намного эпичней родстера на марс , который проебали, а по сложности сопоставимая задача, ну может чуточку сложнее. Можно ещё бахнуть где-то на полпути, а вдруг в труху.
>>8320
А если бы на луне орбиту сместило? Или породу на Землю в виде метеоритного дождя занесло?
398 458323
>>8320
Совки кстати тоже хотели ебнуть по луне.

>Сейчас я вот кстати думаю: если бы они все таки это сделали тогда, то как сегодня выглядела бы теория лунного заговора? Ничего не было, ядерный взрыв на луне снят в павильоне?


Ты че ебнутый? Скинуть бомбу даже проще чем вывести спутник или автоматический посадочный аппарат, а в этом даже отборные дегенераты не сомневались.
image.png99 Кб, 277x182
399 458325
>>8323

>Скинуть бомбу даже проще чем вывести спутник


Но если на Венеру, то не все так однозначно. Задачи научные ведь тоже есть. Первая попытка терраформирования, например.
400 458326
>>8322
Ну на самом деле что бы сместить луну с орбиты на какую-то серьезную величину нужна бомба такой эпической мощности, что после детонации большей части жизни на земле будет уже все равно.

Про породу опасения справедливые, но опять же - отколоть серьезный кусок даже царь-бомбой крайне сложно. Плюс луна, сука, медленная по сравнению с большинством сближающихся с землёй астероидов. А если ты вспомнишь формулу кинетической энергии - то там участвует масса в первой степени, но целый квадрат скорости. Поэтому особой угрозы от гипотетических осколков нет.

Посмотри онемэ, "Ковбой Бибоп" называется. Там есть пара серий где сравнительно реалистично показан быт землян после катастрофы, когда луну рампидорасило.
401 458328
>>8323

>


Ты че ебнутый? Скинуть бомбу даже проще чем вывести спутник или автоматический посадочный аппарат, а в этом даже отборные дегенераты не сомневались.

Ну знаешь, нельзя дегенератов недооценивать. В конце концов плоскоземельцам ничего не мешает процветать и даже копить деньги на круиз краю Земли :(
402 458331
>>8326

>Ковбой Бибоп


Посмотрел и осуждаю. Антинаука там везде. Не топчик, и даже не топчик аниме про космос, но японцы действительно старались. Идея кстате такая, что по венере можно хоть 200, хоть 300 мегатонн - на Земле будет заметен один только пук, а от ебейшего взрыва на луне реально люди могут пострадать. Интересно ещё как будет себя взрывная волна С02 при 80 атм вести. Понятно, что груза будет тонн на 20-30, то и на 50 со всем научным барахлом. Можно с орбиты Земли всё это дело пускать, туда по частям завозить. Дорого это конечно будет. Интересно, есть ли шансы на космические полёты со старта с летящего самолёта. Блю Орижин вроде когда-то "успешен" был, хоть понятно что суборбита по параболе не орбита. Но ведь он достаточно маленький. Если с гиганта уровня "Мрии" пускать что-то, то возможно это будет достаточно баков горючки чтобы полезного в тру космос выводить.
403 458332
>>8322

>А если бы на луне орбиту сместило?


Мощности даже близко не те.

>Или породу на Землю в виде метеоритного дождя занесло?


Для этого надо бомбу заглубить, что проблематично сделать не на Земле. Опять же, подозреваю, что мощности нужны большие на пару порядков. Особенно учитывая, что 99,9% полетит мимо, а остальное сгорит в атмосфере.

На самом деле, на Луне вышел бы пук. Без атмосферы все эти взрывы резко теряют в эпичности. Была бы еле заметная с Земли вспышка на пару секунд (в зависимости от мощности), и тащемта все. Взрывной кратер и оплавленную породу один хуй без телескопа не увидишь.
404 458333
>>8332
Люди уже были на луне, там если и взрывать, то нужно возвращаться с пилотируемой миссией, буром и ЯО. Бурить скважину метров 100-150, ложить заряд, возможно даже "цементировать" ещё (непонятно чем?). Брюс_Уиллис_наносит_ответный_удар_Армагедону.жпг
Вот у Венеры атмосферы углекислоты столько, что гореть будет как новогодняя ёлка, и по другому ту атмосферу и поверхность сложно по всей планете изучить. Вот если бахнуть с побитием атмосферы...
image.png1,2 Мб, 1280x720
405 458402
>>8333

>Вот у Венеры атмосферы углекислоты столько, что гореть будет как новогодняя ёлка

406 458405
>>8402
Элементарно, с образованием CO3 и выпадением C2!
407 458406
>>8405
СО3 с чем связывать собрался?
408 458407
>>8406
В венерианских условиях он стабилен. А если жахнуть супербомбой, то вместо C2 вообще выпадут алмазы, можно потом летать и собирать.
409 458474
Если всех людей собрать, и выкинуть в открытый космос ,в действительно пустое пространство , быть может за пределы галактики даже. Их массы хватит что бы сформировать огромный ледяной комок?
410 458478
>>8474
Это не зависит от массы, в пустом пространстве столкнутся и слипнутся даже две песчинки.
411 458479
>>8478
то есть получится полноценная ледяная человеческая комета? которая будет лететь и собирать вокруг себя остальную пыль, газы?
412 458481
>>8479
Слегка подтухшая, правда. И в межгалактическом пространстве собирать особо нечего, вся не скрытая масса давно собралась в галактики.
413 458486
>>8481

> вся


Всего-то больше половины барионов летают в виде газа между галактик. Понятно что плотность не та, но "вся" - не то слово.
414 458487
>>8481

>подтухшая


Так разве бактерии не отъедут вместе с носителем практически сразу?
415 458488
>>8486
Даже не больше половины а 85%.
416 458489
>>8486
Это не доказано
417 458491
>>8489
Доказано. И симуляции, и прямые наблюдения подтверждают это.
418 458497
Если ток это направленный поток заряженных частиц, то почему в проводнике по которому течёт ток не кончаются электроны? переходят ли электроны из одного проводника в другой при, например, скрутке?
419 458498
>>8497
Электроны в проводе не текут, как вода в трубе. Они просто толкают соседние электроны, а те в свою очередь толкают следующие.
420 458499
>>8498
А вода блять не так?
421 458500
>>8481
можем провести эксперимент - направь член себе в рот и нассы - это движение молекул воды.
6fa92ce1-e5f3-4d8f-a4ac-e77294da0f9f.jpg67 Кб, 510x510
422 458509
>>8499
Нет, блядь, не так.
423 458522
>>8407
А доп. кислород для горения СО2 где возьмёшь?
424 458523
>>8487
Внутри им норм будет в первое время.
425 458525
>>8498
В проводе текут не электроны, а "ударная волна" их смещения.
426 458528
>>8525
Ну я примерно так и написал. Алсо электроны таки текут, насколько я помню, но крайне слоупочно.
427 458551
если ток это когда электроны толкают, то почему мы платим за электричество, все на месте же, в розеточке!
428 458558
>>8497
Вангую, что кончаются, просто их дохуя. "Батарейка села", подозреваю, это в некотором роде оно, хоть там и химические реакции вроде как. Плюс, электроны могут банально производиться на источнике, хуй там эти квантовые поля разберешь.

Ну а если у тебя переменный ток, как в 99% случаев, то там электроны никуда не перемещаются, а просто колбасятся на месте туда-сюда.
429 458563
>>8558
Разность потенциалов же. В случае с батарейкой - натурально кончаются электроны. На отрицательном электроде их много, на положительном мало, отсюда разность. В процессе разряда потихоньку перетекают с минуса на плюс. В случае переменного тока просто болтаются туда-сюда как дебилы.
430 458582
>>8528
Эсли переменка, не текут.
431 458597
А почему не практикуют дозаправку в космосе? В смысле вот одной ракетой ты выводим большую тупую канистру с топливом, второй ракетой большой умный КА с большим но пустым топливным баком. На орбите они стыкуются и солярка перекачивается в КА и ухбля он сразу летит на Плутон. Тогда можно не ебаться со сверхтяжами и обойтись средними недорогим ракетами. Чому так нельзя?
432 458598
>>8597
Геморроя много (и возможностей для отказа и срыва миссии), плюс одна большая ракета, как правило, дешевле кучи мелких. Но Маск вот планирует именно так людишек на Марс пулять.
433 458631
Почему люди не вкладываются в физическое бессмертие? Лучше бы этот вопрос сначала решили, прежде чем тратить огромные суммы на долбанный космос.
434 458635
>>8631
Людей и так слишком много.
435 458638
>>8631
Ну прожил ты лет 300, попробовал всё/перестрадал всё. Что дальше?
436 458639
>>8635
Можно за счет кастрации. Типа хосешь бессмертие - гони яички.
437 458640
>>8638
Можно полететь в космос. Но с бессмертием можно лететь на говне без жизнеобеспечения.
438 458644
>>8639
Ты же понимаешь что найдутся жулики которые красиво обойдут этот закон? К тому же даже если бессмертие станет реальностью то будет уделом элиты. А несменяемость власти это путь в никуда.
>>8640
Это уже электронное бессмертие, тут куча спорных моментов.
439 458645
>>8631
Потому что полеты в космос реальны дают ощутимый выхлоп в виде спутников и прочих осязаемых технологий. Так-то в биоисследования тоже вкладываются, просто нет вау-фактора (и охуенно заметных результатов тоже, только постепенные улучшалки), поэтому ты об этом и не знаешь.
440 458650
А скорость света стопудово постоянна? Типа пространство же расширяется, могла ли раньше скорость света быть другой, пусть даже на полшишечки?
441 458655
>>8644
Будет специальная полиция, которая будет отстреливать яички нарушителям.
442 458675
Есть ли запахи в космосе? Со звуком понятно, но ведь запахи переносятся частицами вещества и по идее должны почувствоваться тем кто рискнет "вдохнуть" вакуум.
443 458681
>>8675
Если "вдохнешь" это вообще как вакуум то будет не до ньюансов ароматических молекул.
444 458721
Я так понимаю, что в целом сегодня сверхзадача века это построить ебический телескоп, найти копию земли более менее в 10-20 световых годах

Дальше запилить искусственную матку, автоматизировать хуе-мое, заморозить 50к оплоготвореных яйцеклеток, роботы там, что бы строили пока пиздюки растут

ну и все это пихнуть в свинцовый гроб вместе с реактором и отправить в 10 000 лет путешествие на 1% от скорости света и молиться, что бы аппаратура пережила 10 000 лет в анабиозе?
445 458727
>>8721

> Я так понимаю


У меня плохие новости. Ты нихуя не понимаешь.
446 458733
>>8675
Можешь залететь в хвост какой-нибудь кометы и вдохнуть сероводорода. Как он пахнет, объяснять не нужно.
447 458758
>>8721
Лучше на марс полететь и жить там
448 458762
>>8758
Доктор Манхэттен, залогиньтесь.
449 458763
>>8762
Слыш, ты с какого райена?
3.jpg276 Кб, 900x900
450 458791
>>8763

>Слыш, ты с какого райена?

451 458797
Можно ли в теории привязать нитку к МКС и опустить другой конец нити на Землю?
452 458799
>>8797
Нитка длиной 400 километров, несущаяся на скорости 27500 км/ч через пространство неравномерной плотности с перепадом давления от нихуя атм до ~1 атм и меняющейся в разные стороны температурой на разных высотах, ну что же может пойти не так?
453 458801
>>8799
И что может пойти не так, зацепится за дерево, или кот обмотается? Можно над окианом повесить.
454 458805
>>8799
Из легкого углеродного волокна да и сама нитка будет не до земли а до уровня облаков примерно. Какие подводные камни? Что будет если ее дернет человек?
455 458827
>>8801
>>8805

>Какие подводные камни?


Порвёт и сожжёт её нахуй, нет таких материалов, из которых можно сделать что-нибудь, что пережило бы такие нагрузки, хоть ты её из хуйдостаниумного алмазолокна сделай.

>Что будет если ее дернет человек?


Он дёрнет себе анус, потому что ему сорвёт мясо с костей ударной волной от несущейся в атмосфере со скоростью 26500 км/ч нитки, ну и ожоги получит заодно.

>Можно над окианом повесить.


Не хотел тебя расстраивать, но МКС не висит на одной точкой поверхности.
456 458832
>>8827
Какая ударная волна, лол. Ее просто сдует ветрами в верхних слоях атмосферы
1248204480456.jpg9 Кб, 251x218
457 458836
Помнится, был сайт, с калькулятором ближайших прохождений заданного спутника, где можно было найти прохождения с наивысшим восхождением. Т.е. вводишь свои координаты, выбираешь спутник и оно выдает результаты на определенный период вперед.
Не могу вспомнить. Дайте ссылку, пжлст!
458 458838
>>8836
Если точнее, выдает даты, время и угол восхождения. Интересны именно они.
1545774780097.jpg42 Кб, 800x800
459 458844
1) Почему объект при наборе скорости, близкой к скорости света, набирает массу, близку к безконечно? Как так? Почему?
Объясните на простом примере как в примере объяснения электричества сраньём с ветки
2) Почему скорость света такая, а не 1 000 000км/ч, и не 1 км/ч?
3) Как квантовая связь может быть больше скорости света?
4) Почему мы, люди, считаем, что Юпитер - бездна газа с железокаменным шариком подшипникам на пару сотен км радиуса в центре в роли ядра? Что если атмосфера Юпитера очень густая, но не больше, скажем, 500км, и мы просто не видим литиево-борной коры планеты?
460 458847
>>8163
самое низкое значение g зафиксировано на горе Уаскаран в Перу (9,7639 м/с²) в 1000 км южнее экватора, а самое большое (9,8337 м/с²) — в 100 км от северного полюса
https://ru.m.wikipedia.org/wiki/Ускорение_свободного_падения
461 458848
>>8479
House that Jack built
462 458850
>>8844
1)Не массу, а импульс. По кочану.
2)По репе.
3) Единственное сообщение, которое может быть отправлено, это случайная последовательность «нулей» и «единиц», — смысла в таком сообщении ровно столько же, сколько в послании, составленном из результатов случайных бросков монетки.
4)Такие модели.
463 458851
Что ощущают космонавты в открытом космосе? Каково первый раз осознавать что ты в бездне?
464 458852
>>8851

>Что ощущают космонавты в открытом космосе?


Ощущают, что >>8850 пидор, раз так отвечает в ТТВ.
465 458856
>>8852
Странно, ощутил то же самое, хотя и нахожусь на земле. Можно ли считать это поводом потребовать надбавку за работу в открытом космосе?
466 458864
>>8844
1) Инерционную массу или импульс. Ща модно массой называть массу покоя, которая очевидно не меняется.
2) Так получилось. Алсо, ты понимаешь, что мульен км/ч был бы ровно такой же "произвольной" величиной, потому что километр и час ни к чему такому фундаментальному не привязаны? Да и 10 - тоже основание от балды.
4) Рискну предположить, что из-за низкой плотности. Масса измеряется по гравитационному влиянию на другие тела, в первую очередь спутники.
Cheeck Pouch.jpg62 Кб, 348x270
467 458883
А могут ли на спутниках Урана и Нептуна быть зеленые и синие ночи из-за отраженного света от планет, вокруг которых они вращаются? Ну чтобы если положить на поверхность белую бумагу она окрашивалась в зеленый и синий цвета соответственно? Если да, это же такой крутой факт, который можно в популяризации использовать.
468 458884
>>8883

> от планет, вокруг которых они вращаются?


Што?
469 458885
>>8884
ну отраженный свет конечно же, просто интересно как именно он отразится и каким ярким будет к примеру Уран и его planetshine для Миранды, если Солнце не будет на горизонте. Хотя у урановских спутников ебанутые ночи длинной в 40 лет с лишним
470 458887
>>8885
Можно прикинуть. Возьмем ближайший к Уран спутник - Миранду - и сравним с освещенностью на Земле отраженным светом Луны.

Луна: 150 млн км от Солнца, 1740 км радиус, альбедо 0,12. До Земли 385 тыс км.

Уран: 2880 млн км от Солнца, 25000 км радиус, альбедо ~0,4. До Миранды 130 тыс км.

Освещенность падает как квадрат расстояния, площадь диска - квадрат радиуса, альбедо входит в первой степени. Посчитав все, получаем, что освещенность Миранды Ураном примерно в 16 раз выше, чем освещенность Земли Луной. Но если взять спутник подальше, то освещенность будет ниже, понятно.
471 458889
>>8887
Ближайший из крупных, то есть.
где мои луны блядь, где мои телескопические камеры ааа сука.jpg423 Кб, 1017x797
472 458890
>>8887
Круто, спасибо. Он же еще и больше размером в небе будет, кольца, мелкие луны вроде Пака. Надеюсь доживу до какой-нибудь миссии к Урану.
Хотя вчера смотрел документ, который будет представляться на 50ой планетной конференции, там предлагают некую миссию под названием Quest к Урану и в нём ни слова про луны. Орбита под 80 градусов и одна широкоугольная камера почти гарантируют, что на Луны даже не взглянут, будут дрочить только на всякую магнетику Урана и его атмосферу. Если НАСА пошлют это дерьмо, то останется надеяться лишь на китайцев.
473 458891
>>8890

>Он же еще и больше размером в небе будет


Пиздец насколько больше. Он почти в 15 раз больше Луны и в 3 раза ближе к Миранде, чем Луна к Земле. Собственно, только из-за этого освещенность такая и получается: он очень тусклый в небе Миранды, но за счет здорового диска светит неплохо так.
474 458892
Есть такая планета, как марс, и на ней есть такая планета как Олимп.
Посмотри на неё, анон. По-моему, эта гора охуела.
Почему у неё настолько отвесные скалы? если это вулкан, то должен быть плавный переход о пика к краям и сама гара должна быть более отстрой. И если про остроту горы ещё можно сказать, что осела от веса или потому что остыла, то с краями всё не ясно.
Почему у Олимпа такие края? Они такие отвесные, будто гору как в мультиках срезали откуда-то с другой планеты, а затем Ctrl+C и Ctrl+V на Маср, забыв заблюрить края в божественном пространственно-временном фотошопе материи.
475 458894
>>8892

> Вулкан имеет крутые склоны по краям высотой до 7 км. Причины образования этих гигантских обрывов пока не нашли убедительного объяснения, хотя многие склоняются к версии подмыва склонов вулкана некогда существовавшим на Марсе океаном.

OlympusMons.jfif852 Кб, 692x1448
476 458895
>>8892

>Есть такая планета...и на ней есть такая планета


Кек лул.
Короткий ответ - хуй знает. Длинный - есть парочка теорий на это счёт. Загибай пальцы:
1. Застывшие лавовые потоки или затвердевшие отложения пепла, которые подсыпались из-за эрозии
2. Подмыло водой, когда на Марсе ещё была вода
3. Подгрызло ледниками, когда на Марсе ещё были йоба-ледники
4. Самая нормальная - это более "хрупкие" породы, которые осыпались при вспучивании коры во время роста вулкана.
477 458896
>>8892
алло мань, марс мало чем от земли в свое время отличался, принимай это как должное, после нормальной экспедиции еще и кости там найдут, можешь скринить
478 458897
>>8892
Бородавка на фейсе поанеты
479 458898
Почему на экваторе восход и закат каждый день все мозже и позже?
480 458899
>>8898
Я так понимаю, ты имел ввиду изменение продолжительности светового дня в течение года? Экватор же не совпадает с плоскостью эклиптики, мы под наклоном 23 градуса хуярим.
https://www.youtube.com/watch?v=9n04SEzuvXo
481 458958
>>8525
>>8498
Есть ссылка где доходчиво этот вопрос описывается?
482 458960
>>8958
Мне это еще училка по физике рассказывала в 7 классе, например.
orig-1517130327fde7a76298fcfd5f8dbdccf276642964.jpeg375 Кб, 750x422
483 458967
Если б на этой картинке были реальные угловые размеры звёзд, как бы это выглядело?
484 458969
>>8967
Вся эта картинка (кусок NGC 3201) - примерно как спичечная головка с двух метров.
485 458981
>>8844
1) Такая физическая модель мира. Шоб её не косоёбило, хуизикам пришлось сделать такое допущение.
2) Скорость света - 1 в любой произвольной си
3) Может быть, а может и не быть. Такая модель, которая пока подтверждается маняэкспериментами.
4) Предположили так на основе имеющихся знаний по планетологии.
Блэт. Уясни уже для себя суть понятия - МОДЕЛЬ.
Электрон, возможно, и не существует вовсе, но модель, использующая сущность "электрон" позволяет предсказать все эффекты, связанные с соответствующими явлениями настолько точно, что обывала может даже, почесав пузо, достаточно уверенно сказать: "электрон - сущность".
486 458983
>>8967
Как квадрат Малевича.
sage 487 458984
>>8981

>модель


>Такая модель


>МОДЕЛЬ


>но модель


>модель? ну мам, модель же! М-О-Д-Е-Л-Ь!


Просто иди нахуй, говно.
488 458989
>>8983
Но он же прав. Философствует, но прав.
489 459004
>>8892
Щитовой вулкан же
Стикер447 Кб, 512x450
490 459005
>>8984
Ты не ахуел, маня?
1548934324812.jpg112 Кб, 880x587
491 459010
Почему я никогда не видел Млечного пути? Так-то по идеи его можно наблюдать из любой точки мира же. Это как-то из-за атмосферы земли связано или что? Где-то что-то застилает вид? Вон на фото всё так чётко и ярко.
492 459013
>>9010
в городе не увидел, ночью в деревне (а если еще и фонари уличные отключат) видно отлично.
Нужно чистое небо, отсутствие источников света и нормальное зрение для full hd
493 459014
>>9010
Потому что засветка в городе. Я когда в Карелии ночевал в палатке всё прекрасно видел, пока не обдвачевался и зрение не посадил.
Он на самом деле достаточно тусклый, его плохо видно, но было очень классно, когда ты припёрся с фонариком на поляну, выключаешь его, лежишь и смотришь в небо и Млечный путь постепенно проявляется, когда глаза ко тьме привыкают (нужно минут 20).
494 459016
>>9010
Во-первых, это фото с большой выдержкой, а не то, что реально можно наблюдать. Даже в идеальных условиях Млечный путь так не выглядит, реально видна просто мутная белесая полоса через все небо. Нет ни деталей, ни тем более цветов, цветное зрение в темноте вообще не работает.

Во-вторых, Млечный путь очень тусклый и любая засветка сразу уничтожает шансы на его наблюдение, надо выезжать подальше от города.
495 459020
А ведь могли бы порофлить и отправить анона к офтальмологу, обмельчал нынче двач
496 459078
Тут в треде неподалеку происходит разговор про запуск кансата на высоту до 2 км. В связи с чем у меня возник вопрос, а как вообще регламентируются и контролируются любительские запуски ракет? Что будет, если анон соберет ракетку, которая подпрыгнет например не на два а на пять км, кто это будет обнаруживать, как будет звучать официальная причина набутыливания анона. И что если анон упорется и соберет например петарду которая сумеет произвести суборбитальный полет - кто и как на это будет реагировать?
497 459079
>>9078
тебя набутылят если даже радиочастоту забьешь а ты про запуск , который может повредить чей нибудь спутник, зонд
498 459080
>>9078
Однажды из-за метеорологической ракеты чуть ядерная война не началась, так что могут набутылить по жесткому
https://twitter.com/Cirincione/status/1089289823544901633

Ну и банально хуй знает, вдруг ты с управлением не справишься и ракета упадет на жилой район или врежется в самолет какой-нибудь, могут очень больно сделать.
499 459081
>>9020
К офтальмологу - это мицар алькор
500 459090
>>9080
Ого. Исчерпывающе.
Получить лицензию на запуск хуйни в космос обычному Васе с автобазы, надо полагать, вообще не реально.
501 459095
>>9080
А правда что если запустить ведро фольги то потом тебя в очко выебут?
502 459111
>>9095
Ты можешь найти и попроще способы быть выебанным в очко, гомик.
503 459119
>>9095
Не пали контору.
504 459146
Пилите перекат, косманы.
505 459156
>>9146
Мы еще даже наполовину вниз не уехали. Пили сам, если хочешь.
506 459162
>>9156
Так пилилка не выросла.
508 459863
>>8838
heavens-above?
Тред утонул или удален.
Это копия, сохраненная 20 июня 2019 года.

Скачать тред: только с превью, с превью и прикрепленными файлами.
Второй вариант может долго скачиваться. Файлы будут только в живых или недавно утонувших тредах. Подробнее

Если вам полезен архив М.Двача, пожертвуйте на оплату сервера.
« /spc/В начало тредаВеб-версияНастройки
/a//b//mu//s//vg/Все доски